Arrest Searches and Seizures2

32
CONSTITUTIONAL LAW II: ARREST, SEARCHES, AND SEIZURES Yee Sue Koy vs. Almeda [GR 47021, 15 June 1940] Laurel (J): 3 concur, 1 concurs in result Facts: In response to a sworn application of Mariano G. Almeda, chief agent of the Anti- Usury Board, dated 5 May 1938, the justice of the peace of Sagay, Occidental Negros, after taking the testimony of applicant's witness, Jose Estrada, special agent of the Anti-Usury Board, issued on the same date a search warrant commanding any peace officer to search during day time the store and premises occupied by Sam Sing & Co., situated at Sagay, Occidental Negros, as well as the person of said Sam Sing & Co., and to seize the documents, notebooks, lists, receipts and promissory notes being used by said Sam Sing & Co. in connection with their activities of lending money at usurious rates of interest in violation of law, or such as may be found, and to bring them forthwith before the aforesaid justice of the peace of Sagay. On the same date, at 10:30 a. m., search was accordingly made by Mariano G. Almeda, Jose Estrada, 2 internal revenue agents and 2 members of the Philippine Army, who seized certain receipt books, vales or promissory notes, chits, notebooks, journal book, and collection list belonging to Sam Sing & Co. and enumerated in the inventory receipt issued by Mariano G. Almeda to the owner of the documents, papers and articles seized. Immediately after the search and seizure thus effected, Mariano G. Almeda filed a return with the justice of the peace of Sagay together. With a request that the office of the Anti-Usury Board be allowed to retain possession of the articles seized for examination, pursuant to section 4 of Act 4109, which request was granted. Under the date of 11 March 1939, Godofredo P. Escalona, counsel for Sam Sing & Co. filed a motion with the Court of First Instance (CFI) of Occidental Negros praying that the search warrant and the seizure effected thereunder be declared illegal and set aside and that the articles in question be ordered returned to Sam Sing & Co., which motion was denied in the order dated 24 July 1939. A similar motion was presented to the justice of the peace of Sagay on 27 October 1939 but was denied the next day. Meanwhile, an information dated 30 September 1939 had been filed in the CFI Occidental Negros, charging Yee Fock alias Yee Sue Koy, Y. Tip and A. Sing, managers of Sam Sing & Co., with a violation of Act 2655. Before the criminal case could be tried, Yee Sue Koy and Yee Tip filed the petition with the Supreme Court on 6 November 1939. The petition is grounded on the propositions (1) that the search warrant issued on 2 May 1938, by the justice of the peace of Sagay and the seizure accomplished thereunder are illegal, because the warrant was issued three days ahead of the application therefor and of the affidavit of the Jose Estrada which is insufficient in itself to justify the issuance of a search warrant, and because the issuance of said warrant manifestly contravenes the mandatory provisions both of section 1, paragraph 3, of Article III of the Constitution and of section 97 of General Orders 58, and (2) that the seizure of the aforesaid articles by means of a search warrant for the purpose of using them as evidence in the criminal case against the accused, is unconstitutional because the warrant thereby becomes unreasonable and amounts to a violation of the constitutional prohibition against compelling the accused to testify against themselves. Issue: Whether the application of the search warrant is supported by the personal knowledge of the witness, besides the applicant, for the judge to determine probable cause in issuing the warrant. Held: Strict observance of the formalities under section 1, paragraph 3, of Article III of the Constitution and of section 97 of General Orders 58 was followed. The applicant Mariano G. Almeda, in his application, swore that "he made his own personal investigation and ascertained that Sam Sing & Co. is lending money without license, charging usurious rate of interest and is keeping, utilizing and concealing in the store and premises occupied by it situated at Sagay, Occidental Negros, documents, notebooks, lists, receipts, promissory notes, and book of accounts and records, all of which are being used by it in connection with its activities of lending money at usurious rate of interest in violation of the Usury Law." In turn, the witness Jose Estrada, in his testimony before the justice of the peace of Sagay, swore that he knew that Sam Sing & Co. was lending money without license and charging usurious rate of interest, because he personally investigated the victims who had secured loans from said Sam Sing & Co. and were charged usurious rate of interest; that he knew that the said Sam Sing & Co. was keeping and using books of accounts and records containing its transactions relative its activities as money lender and the entries of the interest paid by its debtors, because he saw the said Sam Sing & d make entries and records of their debts and the interest paid thereon. As both Mariano G. Almeda and Jose Estrada swore that they had personal knowledge, their affidavits were sufficient for, thereunder, they could be held liable for perjury if the facts would turn out to be not as their were stated under oath. That the existence of probable cause had been determined by the justice of the peace of Sagay before issuing the search warrant complained of, is shown by the following statement in the warrant itself, to wit: "After examination under oath of the complainant, Mariano G. Almeda, Chief Agent

description

Arrest Searches and Seizures2

Transcript of Arrest Searches and Seizures2

CONSTITUTIONAL LAW II: ARREST, SEARCHES, AND SEIZURES

Yee Sue Koy vs. Almeda [GR 47021, 15 June 1940]

Laurel (J): 3 concur, 1 concurs in result

Facts: In response to a sworn application of Mariano G. Almeda, chief agent of the Anti-Usury Board, dated 5 May 1938, the justice of the peace of Sagay, Occidental Negros, after taking the testimony of applicant's witness, Jose Estrada, special agent of the Anti-Usury Board, issued on the same date a search warrant commanding any peace officer to search during day time the store and premises occupied by Sam Sing & Co., situated at Sagay, Occidental Negros, as well as the person of said Sam Sing & Co., and to seize the documents, notebooks, lists, receipts and promissory notes being used by said Sam Sing & Co. in connection with their activities of lending money at usurious rates of interest in violation of law, or such as may be found, and to bring them forthwith before the aforesaid justice of the peace of Sagay. On the same date, at 10:30 a. m., search was accordingly made by Mariano G. Almeda, Jose Estrada, 2 internal revenue agents and 2 members of the Philippine Army, who seized certain receipt books, vales or promissory notes, chits, notebooks, journal book, and collection list belonging to Sam Sing & Co. and enumerated in the inventory receipt issued by Mariano G. Almeda to the owner of the documents, papers and articles seized. Immediately after the search and seizure thus effected, Mariano G. Almeda filed a return with the justice of the peace of Sagay together. With a request that the office of the Anti-Usury Board be allowed to retain possession of the articles seized for examination, pursuant to section 4 of Act 4109, which request was granted. Under the date of 11 March 1939, Godofredo P. Escalona, counsel for Sam Sing & Co. filed a motion with the Court of First Instance (CFI) of Occidental Negros praying that the search warrant and the seizure effected thereunder be declared illegal and set aside and that the articles in question be ordered returned to Sam Sing & Co., which motion was denied in the order dated 24 July 1939. A similar motion was presented to the justice of the peace of Sagay on 27 October 1939 but was denied the next day. Meanwhile, an information dated 30 September 1939 had been filed in the CFI Occidental Negros, charging Yee Fock alias Yee Sue Koy, Y. Tip and A. Sing, managers of Sam Sing & Co., with a violation of Act 2655. Before the criminal case could be tried, Yee Sue Koy and Yee Tip filed the petition with the Supreme Court on 6 November 1939. The petition is grounded on the propositions (1) that the search warrant issued on 2 May 1938, by the justice of the peace of Sagay and the seizure accomplished thereunder are illegal, because the warrant was issued three days ahead of the application therefor and of the affidavit of the Jose Estrada which is insufficient in itself to justify the issuance of a search warrant, and because the issuance of said warrant manifestly contravenes the mandatory provisions both of section 1, paragraph 3, of

Article III of the Constitution and of section 97 of General Orders 58, and (2) that the seizure of the aforesaid articles by means of a search warrant for the purpose of using them as evidence in the criminal case against the accused, is unconstitutional because the warrant thereby becomes unreasonable and amounts to a violation of the constitutional prohibition against compelling the accused to testify against themselves.

Issue: Whether the application of the search warrant is supported by the personal knowledge of the witness, besides the applicant, for the judge to determine probable cause in issuing the warrant.

Held: Strict observance of the formalities under section 1, paragraph 3, of Article III of the Constitution and of section 97 of General Orders 58 was followed. The applicant Mariano G. Almeda, in his application, swore that "he made his own personal investigation and ascertained that Sam Sing & Co. is lending money without license, charging usurious rate of interest and is keeping, utilizing and concealing in the store and premises occupied by it situated at Sagay, Occidental Negros, documents, notebooks, lists, receipts, promissory notes, and book of accounts and records, all of which are being used by it in connection with its activities of lending money at usurious rate of interest in violation of the Usury Law." In turn, the witness Jose Estrada, in his testimony before the justice of the peace of Sagay, swore that he knew that Sam Sing & Co. was lending money without license and charging usurious rate of interest, because he personally investigated the victims who had secured loans from said Sam Sing & Co. and were charged usurious rate of interest; that he knew that the said Sam Sing & Co. was keeping and using books of accounts and records containing its transactions relative its activities as money lender and the entries of the interest paid by its debtors, because he saw the said Sam Sing & d make entries and records of their debts and the interest paid thereon. As both Mariano G. Almeda and Jose Estrada swore that they had personal knowledge, their affidavits were sufficient for, thereunder, they could be held liable for perjury if the facts would turn out to be not as their were stated under oath. That the existence of probable cause had been determined by the justice of the peace of Sagay before issuing the search warrant complained of, is shown by the following statement in the warrant itself, to wit: "After examination under oath of the complainant, Mariano G. Almeda, Chief Agent of the Anti-Usury Board, Department of Justice and Special Agent of the Philippine Army, Manila, and the witness he presented, . . . and this Court, finding that there is just and probable cause to believe as it does believe, that the above described articles, relating to the activities of said Sam Sing & Co. of lending money at usurious rate of interest, are being utilized and kept and concealed at its store and premises occupied by said Sam Sing & Co., all in violation of law."

Alvarez vs. Court of First Instance of Tayabas [GR 45358, 29 January 1937]

First Division, Imperial (J): 4 concur

Facts: On 3 June 1936, the chief of the secret service of the Anti-Usury Board, of the Department of Justice, presented to Judge Eduardo Gutierrez David then presiding over the Court of First Instance of Tayabas, an affidavit alleging that according to reliable information, Narciso Alvarez kept in his house in Infanta, Tayabas, books, documents, receipts, lists, chits and other papers used by him in connection with his activities as a moneylender, charging usurious rates of interest in violation of the law. In his oath at the end of the affidavit, the chief of the secret service stated that his answers to the questions were correct to the best of his knowledge and belief. He did not swear to the truth of his statements upon his own knowledge of the facts but upon the information received by him from a reliable person. Upon the affidavit the judge, on said date, issued the warrant which is the subject matter of the petition, ordering the search of the Alvarez's house at any time of the day or night, the seizure of the books and documents and the immediate delivery thereof to him to be disposed of in accordance with the law. With said warrant, several agents of the Anti-Usury Board entered Alvarez's store and residence at 7:00 p.m. of 4 June 1936, and seized and took possession of the following articles: internal revenue licenses for the years 1933 to 1936, 1 ledger, 2 journals, 2 cashbooks, 9 order books, 4 notebooks, 4 check stubs, 2 memorandums, 3 bankbooks, 2 contracts, 4 stubs, 48 stubs of purchases of copra, 2 inventories, 2 bundles of bills of lading, 1 bundle of credit receipts, 1 bundle of stubs of purchases of copra, 2 packages of correspondence, 1 receipt book belonging to Luis Fernandez, 14 bundles of invoices and other papers, many documents and loan contracts with security and promissory notes, 504 chits, promissory notes and stubs of used checks of the Hongkong & Shanghai Banking Corporation (HSBC). The search for and seizure of said articles were made with the opposition of Alvarez who stated his protest below the inventories on the ground that the agents seized even the originals of the documents. As the articles had not been brought immediately to the judge who issued the search warrant, Alvarez, through his attorney, filed a motion on 8 June 1936, praying that the agent Emilio L. Siongco, or any other agent, be ordered immediately to deposit all the seized articles in the office of the clerk of court and that said agent be declared guilty of contempt for having disobeyed the order of the court. On said date the court issued an order directing Siongco to deposit all the articles seized within 24 hours from the receipt of notice thereof and giving him a period of 5 days within which to show

CONSTITUTIONAL LAW II: ARREST, SEARCHES, AND SEIZURES

cause why he should not be punished for contempt of court. On 10 June, Attorney Arsenio Rodriguez, representing the Anti-Usury Board, filed a motion praying that the order of the 8th of said month be set aside and that the Anti-Usury Board be authorized to retain the articles seized for a period of 30 days for the necessary investigation. On June 25, the court issued an order requiring agent Siongco forthwith to file the search warrant and the affidavit in the court, together with the proceedings taken by him, and to present an inventory duly verified by oath of all the articles seized. On July 2, the attorney for the petitioner filed a petition alleging that the search warrant issued was illegal and that it had not yet been returned to date together with the proceedings taken in connection therewith, and praying that said warrant be cancelled, that an order be issued directing the return of all the articles seized to Alvarez, that the agent who seized them be declared guilty of contempt of court, and that charges be filed against him for abuse of authority. On September 10, the court issued an order holding: that the search warrant was obtained and issued in accordance with the law, that it had been duly complied with and, consequently, should not be cancelled, and that agent Siongco did not commit any contempt of court and must, therefore, be exonerated, and ordering the chief of the Anti-Usury Board in Manila to show cause, if any, within the unextendible period of 2 days from the date of notice of said order, why all the articles seized appearing in the inventory should not be returned to Alvarez. The assistant chief of the Anti-Usury Board of the Department of Justice filed a motion praying, for the reasons stated therein, that the articles seized be ordered retained for the purpose of conducting an investigation of the violation of the Anti-Usury Law committed by Alvarez. On October 10, said official again filed another motion alleging that he needed 60 days to examine the documents and papers seized, which are designated on pages 1 to 4 of the inventory by Nos. 5, 10, 16, 23, 25-27, 30-31 , 34, 36-43 and 45, and praying that he be granted said period of 60 days. In an order of October 16, the court granted him the period of 60 days to investigate said 19 documents. Alvarez, herein, asks that the search warrant as well as the order authorizing the agents of the Anti-Usury Board to retain the articles seized, be declared illegal and set aside, and prays that all the articles in question be returned to him.

Issue: Whether the search warrant issued by the court is illegal because it has been based upon the affidavit of agent Almeda in whose oath he declared that he had no personal knowledge of the facts which were to serve as a basis for the issuance of the warrant but that he had knowledge thereof through mere information secured from a person whom he considered reliable, and that it is illegal as it was not supported by other affidavits aside from that made by the applicant.

Held: Section 1, paragraph 3, of Article III of the Constitution and Section 97 of General Orders 58 require that there be not only probable cause before the issuance of a search warrant but that the search warrant must be based upon an application supported by oath of the applicant and the witnesses he may produce. In its broadest sense, an oath includes any form of attestation by which a party signifies that he is bound in conscience to perform an act faithfully and truthfully; and it is sometimes defined as an outward pledge given by the person taking it that his attestation or promise is made under an immediate sense of his responsibility to God. The oath required must refer to the truth of the facts within the personal knowledge of the petitioner or his witnesses, because the purpose thereof is to convince the committing magistrate, not the individual making the affidavit and seeking the issuance of the warrant, of the existence of probable cause. The true test of sufficiency of an affidavit to warrant issuance of a search warrant is whether it has been drawn in such a manner that perjury could be charged thereon and affiant be held liable for damages caused. The affidavit, which served as the exclusive basis of the search warrant, is insufficient and fatally defective by reason of the manner in which the oath was made, and therefore, the search warrant and the subsequent seizure of the books, documents and other papers are illegal. Further, it is the practice in this jurisdiction to attach the affidavit of at least the applicant or complainant to the application. It is admitted that the judge who issued the search warrant in this case, relied exclusively upon the affidavit made by agent Almeda and that he did not require nor take the deposition of any other witness. Neither the Constitution nor General Orders 58 provides that it is of imperative necessity to take the depositions of the witnesses to be presented by the applicant or complainant in addition to the affidavit of the latter. The purpose of both in requiring the presentation of depositions is nothing more than to satisfy the committing magistrate of the existence of probable cause. Therefore, if the affidavit of the applicant or complainant is sufficient, the judge may dispense with that of other witnesses. Inasmuch as the affidavit of the agent was insufficient because his knowledge of the facts was not personal but merely hearsay, it is the duty of the judge to require the affidavit of one or more witnesses for the purpose of determining the existence of probable cause to warrant the issuance of the search warrant. When the affidavit of the applicant or complainant contains sufficient facts within his personal and direct knowledge, it is sufficient if the judge is satisfied that there exists probable cause; when the applicant's knowledge of the facts is mere hearsay, the affidavit of one or more witnesses having a personal knowledge of the facts is necessary. Thus the warrant issued is likewise illegal because it was based only on the affidavit of the agent who had no personal knowledge of the facts.

Mata vs. Bayona [GR 50720, 26 March 1984]

Second Division, de Castro (J): 3 concur, 2 concur in result, 1 took no part

Facts: Soriano Mata was accused under Presidential Decree (PD) 810, as amended by PD 1306, the information against him alleging that Soriano Mata offered, took and arranged bets on the Jai Alai game by "selling illegal tickets known as 'Masiao tickets' without any authority from the Philippine Jai Alai & Amusement Corporation or from the government authorities concerned." Mata claimed that during the hearing of the case, he discovered that nowhere from the records of the said case could be found the search warrant and other pertinent papers connected to the issuance of the same, so that he had to inquire from the City Fiscal its whereabouts, and to which inquiry Judge Josephine K. Bayona, presiding Jufe of the City Court of Ormoc replied, "it is with the court". The Judge then handed the records to the Fiscal who attached them to the records. This led Mata to file a motion to quash and annul the search warrant and for the return of the articles seized, citing and invoking, among others, Section 4 of Rule 126 of the Revised Rules of Court. The motion was denied by the Judge on 1 March 1979, stating that the court has made a thorough investigation and examination under oath of Bernardo U. Goles and Reynaldo T. Mayote, members of the Intelligence Section of 352nd PC Co./Police District II INP; that in fact the court made a certification to that effect; and that the fact that documents relating to the search warrant were not attached immediately to the record of the criminal case is of no moment, considering that the rule does not specify when these documents are to be attached to the records. Mata's motion for reconsideration of the aforesaid order having been denied, he came to the Supreme Court, with the petition for certiorari, praying, among others, that the Court declare the search warrant to be invalid for its alleged failure to comply with the requisites of the Constitution and the Rules of Court, and that all the articles confiscated under such warrant as inadmissible as evidence in the case, or in any proceedings on the matter.

Issue: Whether the judge must before issuing the warrant personally examine on oath or affirmation the complainant and any witnesses he may produce and take their depositions in writing, and attach them to the record, in addition to any affidavits presented to him.

Held: Under the Constitution "no search warrant shall issue but upon probable cause to be determined by the Judge or such other responsible officer as may be authorized by law after examination under oath or affirmation of the complainant and the witnesses

CONSTITUTIONAL LAW II: ARREST, SEARCHES, AND SEIZURES

he may produce". More emphatic and detailed is the implementing rule of the constitutional injunction, The Rules provide that the judge must before issuing the warrant personally examine on oath or affirmation the complainant and any witnesses he may produce and take their depositions in writing, and attach them to the record, in addition to any affidavits presented to him. Mere affidavits of the complainant and his witnesses are thus not sufficient. The examining Judge has to take depositions in writing of the complainant and the witnesses he may produce and to attach them to the record. Such written deposition is necessary in order that the Judge may be able to properly determine the existence or nonexistence of the probable cause, to hold liable for perjury the person giving it if it will be found later that his declarations are false. We, therefore, hold that the search warrant is tainted with illegality by the failure of the Judge to conform with the essential requisites of taking the depositions in writing and attaching them to the record, rendering the search warrant invalid.

OLAES V. PEOPLE

Olaes was indicted for violation of Dangerous Drugs Act of 1972. He was believed to have in his possession marijuana dried stalks/leaves/seeds/cigarettes and other regulated/prohibited and exempt narcotics preparations.

Facts:

1.AdolfoOlaes was believed to have in his possession marijuana dried stalks/leaves/seeds/cigarettes and other regulated/prohibited and exempt narcotics preparations; thus, indicting petitioners of violation of RA 6425(Dangerous Drugs Acts of 1972) despite failure to pinpoint specific section of same.

2.Petitioners challenged the admission of evidence seized by virtue of an allegedly invalid warrant issued on March. More so, petitioners claimed that the search warrant issued by the judge is unconstitutional because it did not indicate the specific offense the petitioners have supposedly committed; thus, making no valid finding of probable cause as a justification for the issuance of the said warrant inconformity with the Bill of Rights.

Ruling:

1. Although the specific section of the Dangerous Drugs Act is not pinpointed, there isno question at all of the specific offense

alleged to have been committed as a basisfor the finding of probable cause. The search warrant also satisfies therequirement in the Bill of Rights of the particularity of the description to be made of the “place to be searched and the persons or things to be seized.”

Thus, thearticles seized under the challenged search warrant were admitted as evidence

NEMESIO PRUDENTE vs Hon Judge ABELARDO M. DAYRIT

FACTS:

The Chief of the Intelligence Special Action Division (ISAD) filed with the Regional Trial Court (RTC) Manila, Judge AbelardoDayrit, for the issuance of Search Warrant for violation of PD No. 1866 (Illegal Possession of Firearm, etc). In the deposition of witness (P/Lt. Florencio C. Angeles), it was made mentioned of “result of our continuous surveillance conducted for several days. We gathered information from verified sources that the holders of said firearms and explosives as well as ammunitions aren’t licensed to possess said firearms and ammunition. Further, the premises is a school and the holders of these firearms are not student who were not supposed to possess firearms, explosives and ammunitions.

Person to be searched in NemesioPrudente at the Polytechnic University of the Philippines, Sta. Mesa, Sampaloc, Manila, has in his control or possession firearms, explosives hand grenades and ammunitions which are illegally possesses at the office of Department of Military Science and Tactics and at the office of the President.

Petitioner moved to quash the Search Warrant. He claimed that:

Petitioners, had no personal knowledge of the facts

The examination of the said witness was not in form of searching questions and answers

Search warrant was a general warrant

Violation of Circular No. 19 of the Supreme Court in that the complainant failed to allege under oath that the issuance of the search warrant on a Saturday, urgent.

ISSUE:

Whether or not the search and seizure was valid?

HELD:

Search Warrant annulled and set aside.

RATIONALE:

Valid search warrant to issue, there must be probable cause, which is to be determined personally by the Judge, after examination under oath and affirmation of the complainant, and that witnesses he may produce and particularly describing the place to be searched and the persons and things to be seized. The probable cause must be in connection with one specific offense and the Judge must, before issuing Search Warrant, personally examine in the form of searching questions and answers, In writing and under oath, the complainant and any witnesses he may produce, on facts personally known to them and attach to the record their sworn statements together with any affidavits submitted.

“Probable Cause” for a valid search warrant, has been defined as such facts and circumstances which would lead a reasonably discreet and prudent man to believe that an offense has been committed, and that objects sought in connection which the offense are in the place sought to be searched.

- This probable case must be shown to be personal knowledge and of the complainant and witnesses he may produce and not based on mere hearsay.

PARTICULARITY

For violation of PD 1866 (Illegal Possession of Firearms, etc.) while the said decree punishes several offenses, the alleged violation in this case was, qualified by the phrase illegal possession of firearms etc. – – Reformed to ammunitions and explosives. In other words, the search warrant was issued for the specific offense of illegal possession of firearms and explosives. Hence, the failure of the Search Warrant to mention the particular provision of PD1-866

CONSTITUTIONAL LAW II: ARREST, SEARCHES, AND SEIZURES

that was violated is not of such gravity as to call for the invalidation of this case.

Chia vs Collector of Customs

FACTS:

A verified report of a confidential informant that asserted electronic and electrical equipment and other articles illegally imported into the Philippines by a syndicate engaged in unlawful “shipside” activities (foreign goods are unloaded form foreign ships in transit through the Bureau of Customs, thereby evading payment of the corresponding customs duties, and were found inside the “Tom’s electronics” and “Sony Merchandising” after valuation, the Collector of Customs issued warrants of seizure and detention Nos: 14925 ad 14925-A.

Search Warrant:

Various electronics equipment like cassette tape recorders, car stereos, phonograph needles, portable TV sets, imported long playing records, spare parts of TVs and Radios and other electrical appliances

ISSUE:

Whether the warrants of seizure and detention are general warrant issued in violation of Rule 126, Sections 3 of Rules of Court?

HELD:

Petition dismissed.

RATIONALE:

Section 2208 (Customs) – Tariff and Customs Code – Right of Police Officer to enter in closure, a warehouse, store, or other building or in closure used for keeping or storage or articles does not become a dwelling house within the meaning.

Section 2536 (Seizure of other articles) – The Commissioner of Customs and Collector of Customs and or any other Customs officer, with the prior authorization in writing by the commissioner, may demand evidence of payment of duties and taxes on foreign articles.

Search of Dwelling House (Section 2209) – Upon warrant issued by a Judge of the Court or such other responsible officers as may be authorized by law, upon sworn application showing probable cause and particularly describing the place to be searched and the person or thing to be seized.

The warrants issued by the Collector of Customs in this case were not general warrant, as erroneously alleged by petitioner for they identified the stores to be seized, described the articles to be seized and specified the provision of the Tariff and Customs Code.

20TH CENTURY FOX FILM CORPORATION

vs.

CA

FACTS:

In a letter-complaint dated August 26, 1985, petitioner 20th Century FoxFilm Corporation through counsel sought the National Bureau of Investigation's (NBI) assistance in the conduct of searches and seizures inconnection with the latter's anti-film piracy campaign. Specifically, theletter-complaint alleged that certain videotape outlets all over MetroManila are engaged in the unauthorized sale and renting out of copyrighted films in videotape form which constitute a flagrant violationof Presidential Decree No. 49 (otherwise known as the Decree on theProtection of Intellectual Property).Acting on the letter-complaint, the NBI

conducted surveillance andinvestigation of the outlets pinpointed by the petitioner and subsequentlyfiled three (3) applications for search warrants. On September 4, 1985, thelower court issued the desired search warrants. The NBI accompanied bythe petitioner's agents, raided the video outlets and seized the itemsdescribed therein. An inventory of the items seized was made and leftwith the private respondents. The lower court later on lifted the 3 searchwarrants and ordered the NBI to return the properties that were seized.

ISSUE:

Whether or not the judge properly lifted the search warrants he issuedearlier upon the application of the National Bureau of Investigation onthe basis of the complaint filed by the petitioner.

RULING:

The Supreme Court dismissed the petition.The NBI agents who acted as witnesses did not have personal knowledgeof the subject matter of their testimony which was the alleged commissionof the offense by the private respondents. Only the petitioner's counselwho was also a witness during the application for the issuance of thesearch warrants stated that he had personal knowledge that theconfiscated tapes owned by the private respondents were pirated tapestaken from master tapes belonging to the petitioner. However, the lowercourt did not give much credence to his testimony in view of the fact thatthe master tapes of the allegedly pirated tapes were not shown to thecourt during the application.Witnesses in the hearing for an application for search warrants must havepersonal knowledge of the subject matter of their testimony as to thealleged commission of the offense. Also, the sear warrant must contain aspecific description of the articles to be seized. General warrants areconstitutionally objectionable.

People vs Choi

Facts: The factual antecedents follow.

On April 27, 1999, Mario P. Nieto, Intelligence Operative of the Economic Intelligence and Investigation Bureau, Department of Finance, applied for a search warrant with the Regional Trial Court (RTC) of Angeles City, Pampanga, Branch 56, against respondent Christopher Choi for violation of Section 168, paragraphs 2 and 3

CONSTITUTIONAL LAW II: ARREST, SEARCHES, AND SEIZURES

(a) and (c), in relation to Section 169 of RA 8293, also known as the Intellectual Property Code.

Mario Nieto contended that Choi indeed has in his possession, control and custody [r]eams and packs of fake Marlboro Red Cigarettes, as well as cardboard cases of fake Marlboro Red Cigarettes (each cardboard case contains two (2) [m]aster [c]ases of Marlboro and each [m]aster case contains fifty (50) reams) being distributed, kept and sold thereat in violation of Section 168, par. 2 and 3 (a) and (c) in relation to Section 169 of R.A. 8293

On the other, Choi contended that Judge Gatbalite committed grave abuse of discretion in refusing to quash the search warrant, arguing that probable cause was not sufficiently established as the examination conducted was not probing and exhaustive and the warrant did not particularly describe the place to be searched. Respondent also prayed that Atty. Bennie Nicdao be prohibited from using as evidence the articles seized by virtue of the search warrant. This was granted by the CA in a decision dated April 10, 2002

Issue: the search warrant is valid

Held: Yes.A search warrant can be issued only upon a finding of probable cause. Probable cause means such facts and circumstances which would lead a reasonably discreet and prudent man to believe that an offense has been committed and that the objects sought in connection with the offense are in the place sought to be searched. 22 The determination of the existence of probable cause requires the following:

(1) the judge must examine the complainant and his witnesses personally;

(2) the examination must be under oath and

(3) the examination must be reduced in writing in the form of searching questions and answers.

The searching questions propounded to the applicant and the witnesses depend largely on the discretion of the judge. Although there is no hard-and-fast rule governing how a judge should conduct his examination, it is axiomatic that the examination must be probing and exhaustive, not merely routinary, general, peripheral, perfunctory or pro-forma. The judge must not simply rehash the contents of the affidavit but must make his own inquiry on the intent and justification of the application. The

questions should not merely be repetitious of the averments stated in the affidavits or depositions of the applicant and the witnesses. If the judge fails to determine probable cause by personally examining the applicant and his witnesses in the form of searching questions before issuing a search warrant, grave abuse of discretion is committed.

The determination of probable cause does not call for the application of rules and standards of proof that a judgment of conviction requires after trial on the merits. As the term implies, probable cause is concerned with probability, not absolute or even moral certainty. The standards of judgment are those of a reasonably prudent man, not the exacting calibrations of a judge after a full-blown trial. No law or rule states that probable cause requires a specific kind of evidence. No formula or fixed rule for its determination exists. Probable cause is determined in the light of conditions obtaining in a given situation. The entirety of the questions propounded by the court and the answers thereto must be considered by the judge.

In this case, aside from the testimony of Sealey, petitioner judge also heard the testimony of applicant Nieto.

Nolasco vs. Cruz Pano, 132 SCRA 152 (1985)

FACTS: Milagros Aguilar-Roque was arrested together with Cynthia Nolasco by the Constabulary Security Group (CSG). Milagros had been wanted as a high ranking officer of the CPP. The arrest took place at 11:30 a.m. of August 6, 1984. At noon of the same day, her premises were searched and 428 documents, a portable typewriter and 2 boxes were seized.

Earlier that day, Judge Cruz Paño issued a search warrant to be served at Aguilar-Roque’s leased residence allegedly an underground house of the CPP/NPA. On the basis of the documents seized, charges of subversion and rebellion by the CSG were filed by but the fiscal’s office merely charged her and Nolasco with illegal possession of subversive materials. Aguilar-Roque asked for suppression of the evidence on the ground that it was illegally obtained and that the search warrant is void because it is a general warrant since it does not sufficiently describe with particularity the things subject of the search and seizure, and that probable cause has not been properly established for lack of searching questions propounded to the applicant’s witness.

ISSUE: WON the search warrant was valid?

HELD:

NO. Section 3, Article IV of the Constitution, guarantees the right of the people to be secure in their persons, houses, papers and effects against unreasonable searches and seizures of whatever nature and for any purpose. It also specifically provides that no Search Warrant shall issue except upon probable cause to be determined by the Judge or such other responsible officer as may be authorized by law, after examination under oath or affirmation of the complainant and the witnesses he may produce, and particularly describing the place to be searched and the things to be seized.

It is at once evident that the foregoing Search Warrant authorizes the seizure of personal properties vaguely described and not particularized. It is an all- embracing description which includes everything conceivable regarding the Communist Party of the Philippines and the National Democratic Front. It does not specify what the subversive books and instructions are; what the manuals not otherwise available to the public contain to make them subversive or to enable them to be used for the crime of rebellion. There is absent a definite guideline to the searching team as to what items might be lawfully seized thus giving the officers of the law discretion regarding what articles they should seize as, in fact, taken also were a portable typewriter and 2 wooden boxes.

It is thus in the nature of a general warrant and infringes on the constitutional mandate requiring particular description of the things to be seized. In the recent rulings of this Court, search warrants of similar description were considered null and void for being too general.

PICOP v. Asuncion, 307 SCRA 253) (1999)

FACTS: On January 25, 1995, Police Chief Inspector Napoleon B.

Pascua applied for a search warrant before the RTC of Quezon

City, stating: 1. That the management of Paper Industries

Corporation of the Philippines, located at PICOP compound, is in

possession or ha[s] in [its] control high powered firearms,

CONSTITUTIONAL LAW II: ARREST, SEARCHES, AND SEIZURES

ammunitions, explosives, which are the subject of the offense, or

used or intended to be used in committing the offense, and

which . . . are [being kept] and conceal[ed] in the premises

described; 2. That a Search Warrant should be issued to enable

any agent of the law to take possession and bring to the described

properties. After propounding several questions to Bacolod, Judge

Maximiano C. Asuncion issued the contested search warrant. On

February 4, 1995, the police enforced the search warrant at the

PICOP compound and seized a number of firearms and explosives.

Believing that the warrant was invalid and the search

unreasonable, the petitioners filed a “Motion to Quash” before

the trial court. Subsequently, they also filed a “Supplemental

Pleading to the Motion to Quash” and a “Motion to

SuppressEvidence.” On March 23, 1995, the RTC issued the first

contested Order which denied petitioners’ motions. On August 3,

1995, the trial court rendered its second contested Order denying

petitioners’ Motion for Reconsideration.

ISSUE: WON the search warrant issued was valid

HELD:The requisites of a valid search warrant are: (1) probable

cause is present; (2) such presence is determined personally by

the judge; (3) the complainant and the witnesses he or she may

produce are personally examined by the judge, in writing and

under oath or affirmation; (4) the applicant and the witnesses

testify on facts personally known to them; and (5) the warrant

specifically describes the place to be searched and the things to

be seized. In the present case, the search warrant is invalid

because (1) the trial court failed to examine personally the

complainant and the other deponents; (2) SPO3 Cicero Bacolod,

who appeared during the hearing for the issuance of the search

warrant, had no personal knowledge that petitioners were not

licensed to possess the subject firearms; and (3)the place to be

searched was not described with particularity.

Yousef Al Ghoul vs. Court of Appeals

GR No.126859, September 4, 2001

Facts: Judge Geronimo S. Mangay, presiding judge of the Regional

Trial Court, National Capital Judicial Region, Branch 125, Kalookan

City, issued search warrants 54-953 and 55-954 for the search and

seizure of certain items in Apartment No. 2 at 154 Obiniana

Compound, Deparo Road, Kalookan City.

On April 1, 1995, the police searched Apartment No. 8, in the

same compound and found one (1) .45 caliber pistol.

Found in Apartment No. 2 were firearms, ammunitions and

explosives.

Petitioners were charged before the Regional Trial Court of

Kalookan City accusing them with illegal possession of firearms,

ammunitions and explosives, pursuant to Presidential Decree No.

1866.6 Thereafter, petitioners were arrested and detained.

Petitioners contend that the search and seizure orders violated

Sections 2 and 3 of the Bill of Rights as well as Section 3 of Rule

126 of the Rules of Court on Criminal Procedure because the place

searched and articles seized were not described with particularity.

They argue that the two-witness requirement under Section 10 of

Rule 126 was ignored when only one witness signed the receipt

for the properties seized during the search, and said witness was

not presented at the trial.

ISSUE: W/N the items described in the warrant were sufficiently

described with particularity.

HELD: As held in PICOP v. Asuncion, the place to be searched

cannot be changed, enlarged nor amplified by the police.

Policemen may not be restrained from pursuing their task with

vigor, but in doing so, care must be taken that constitutional and

legal safeguards are not disregarded. Exclusion of unlawfully

seized evidence is the only practical means of enforcing the

constitutional injunction against unreasonable searches and

seizures. Hence, we are constrained to declare that the search

made at Apartment No. 8 is illegal and the .45 caliber pistol taken

thereat is inadmissible in evidence against petitioners.

Now, in contrast, the search conducted at Apartment No. 2 could

not be similarly faulted. The search warrants in question

specifically mentioned Apartment No. 2. The search was done in

the presence of its occupants, herein petitioners, in accordance

with Section 7 of Rule 126, Revised Rules of Court. Petitioners

allege lack of particularity in the description of objects to be

seized pursuant to the warrants. That the articles seized during

the search of Apartment No. 2 are of the same kind and nature as

those items enumerated in the search warrant appears to be

beyond cavil. The items seized from Apartment No. 2 were

described with specifity in the warrants in question. The nature of

the items ordered to be seized did not require a technical

description. Moreover, the law does not require that the things

to be seized must be described in precise and minute details as

to leave no room for doubt on the part of the searching

authorities, otherwise, it would be virtually impossible for the

applicants to obtain a search warrant as they would not know

exactly what kind of things they are looking for. Substantial

similarity of those articles described as a class or species would

suffice.

The case of Bache and Co. (Phil.), Inc. v. Ruiz , 37 SCRA 823, 835

(1971), pointed out that one of the tests to determine the

particularity in the description of objects to be seized under a

search warrant is when the things described are limited to those

which bear direct relation to the offense for which the warrant is

being issued. A careful examination of the Search Warrants shows

that they were worded in such a manner that the enumerated

items to be seized could bear a direct relation to the offense of

violation of Section 1 and 3 of Presidential Decree No.1866, as

CONSTITUTIONAL LAW II: ARREST, SEARCHES, AND SEIZURES

amended, penalizing illegal possession of firearms, ammunitions

and explosives.DEL ROSARIO y NICOLAS vs. PEOPLE,

G.R. No. 142295

PARDO,J

FACTS: Accused-appellant Vicente del Rosario was found guilty of violation of P. D. No. 1866 of the Regional Trial Court of Malolos. Allegedly, sometime in May 1996, the police received a report that accused-appellant Vicente del Rosario was in possession of certain firearms without the necessary licenses. Acting upon the report, the PNP Criminal Investigation Group inquired from the PNPFirearms and Explosive Division whether or not the report was true. The PNP Firearms and Explosives Division issued a certificationstating that per records in his office, the appellant is not a licensed/registered firearm holder of any kind and caliber. Armed with thesaid certificationthe police applied for a search warrant to enable them to search the house of appellant.Upon the issuance of the warrant, a team led by P/Sr. Insp. Adique went to Norzagaray to serve the warrant. Before proceeding tothe residence of the appellant, the police officers requested Barangay Chairman Rogelio de Silva and Barangay Councilman AurelioPanteleon to accompany them in the implementation of the warrant. Upon arrival at the house of appellant, the police officersintroduced themselves to the wife of appellant. When the appellant came out, P/Sr. Insp. Adique informed him that they had asearch warrant and that they were authorized to search his house. After appellant gave his permission, the police officers conducteda search of the house. The search yielded the following items: (a) a caliber .45 pistol with Serial No. 703792 with five magazines of caliber .45 (Exhibits B and H) found at the master's bedroom; (b) five magazines of 5.56 M-16 rifle and two radios (Exhibits C to C-4)found in the room of appellant's daughter; and (c) a caliber .22 revolver with Serial No. 48673 (Exhibit F) containing 8 pieces of liveammunition (Exhibit M) found in the kitchen of the house. When asked about his license to possess the firearms, the appellant failedto produce any. This prompted the police officers to seize the subject firearms.For his defense, appellant contends that he had a license for the caliber .45 pistol recovered in his bedroom and that the

other itemsseized during the search including the caliber .22 revolver, were merely planted by the police officers. Appellant likewise assails themanner in which the search was carried out, claiming that the police officers just barged into his house without asking permission.Furthermore, he claimed that the barangay officials arrived only after the police already had finished the search. However, after trialthe trial court rendered a judgment of conviction which decision was affirmed by the Court of Appeals.

ISSUE: Whether or not the seizure of items not mentioned in the search warrant was illegal.

HELD:The Supreme CourtREVERSESthe decision of the Court of Appeals andACQUITSpetitioner Vicente del Rosario y Nicolas of thecharge of violation of P. D. No. 1866.Seizure is limited to those items particularly described in a valid search warrant. Searching officers are without discretion regardingwhat articles they shall seize. Evidence seized on the occasion of such an unreasonable search and seizure is tainted and excludedfor being the proverbial "fruit of a poisonous tree." In the language of the fundamental law, it shall be inadmissible in evidence for any purpose in any proceedingIn this case, the firearm was not found inadvertently and in plain view. It was found as a result of a meticulous search in the kitchenof petitioner's house. This firearm, to emphasize, was not mentioned in the search warrant. Thus, the seizure is illegal.True that as an exception, the police may seize without warrant illegally possessed firearm or any contraband for that matter,inadvertently found in plain view. However, "[t]he seizure of evidence in 'plain view' applies only where the police officer is notsearching for evidence against the accused, but inadvertently comes across an incriminating object."52Specifically, seizure of evidence in "plain view" is justified when there is:(a) a prior valid intrusion based on the valid warrantless arrest in which the police are legally present in the pursuit of their official duties;(b) the evidence was inadvertently discovered by the police who had the right to be where they are.(c) the evidence must be immediately apparent, and(d) "plain view" justified mere seizure of evidence without further search.

People vs Francisco

FACTS: A Federico Verona and his live-in girlfriend, accused-appellant Annabelle Francisco, were placed under surveillance after the police confirmed, through a test-buy operation, that they were engaged in selling shabu or methamphetamine hydrochloride. SPO2 Teneros and SPO4 Alberto San Juan of OADDI-WPDC, U.N. Avenue, Manila applied for a search warrant before Branch 23 of the Regional Trial Court of Manila to authorize them to search the premises at 122 M. Hizon St., Caloocan City.

Attached to the application was the After-Surveillance Report[1] of SPO2 Teneros. It stated that Dante Baradilla, of 1726 Lallana St., corner Sta. Catalina St., Tondo, Manila, who claimed to be one of Federico Verona’s runners in the illegal drugs operations, allegedly sought the assistance of SPO2 Teneros for the arrest of Verona.[2] The search warrant[3] was subsequently issued by Judge Bayhon authorizing the search of shabu and paraphernalia at No. 122 M. Hizon Street, Caloocan City.

Accused-appellant Annabelle Francisco, who was then nine months pregnant, was resting inside the second floor master’s bedroom of their two- storey apartment at No. 120 M. Hizon Street, Caloocan City, when she heard a loud bang downstairs as if somebody forcibly opened the front door. Eight policemen suddenly entered her bedroom and conducted a search for about an hour. Accused-appellant inquired about their identities but they refused to answer. It was only at the police station where she found out that the team of searchers was led by SPO2 Teneros. The police team, along with Barangay Chairwoman MiguelitaLimpo and Kagawad Bernie de Jesus, both of Barangay 64, Zone-6, District 2, Caloocan City, enforced the warrant and seized the following:[4]

1. One (1) Salad Set marked Pyrex wrapped in a plastic containing white crysthalline (sic) substance or methamphetamine hydrochloride or shabu with markings by the undersigned inside the house of subjects’ residence weighing (230) two hundred thirty (sic) grams of methampetamine hydrochloride or shabu by Aida Abear-Pascual of NBI Forensic Chemist;

2. Several plastics in different sizes;

3. Two (2) roll of strip aluminum foil;

CONSTITUTIONAL LAW II: ARREST, SEARCHES, AND SEIZURES

4. Five (5) tooter water pipe and improvised and two burner improvised;

5. Two (2) pantakal or measuring weight in shabu;

6. Two (2) cellular phone motorola with markings;

7. One (1) monitoring device with cord and with markings;

8. Several pcs. with strip aluminum foil;

9. Two (2) masking tip (sic) with markings;

10. Twentee (sic) two thousand nine hundred ninetee (sic) pesos.

The police team also allegedly seized the amount of P180,000.00, a Fiat car, jewelry, set of keys, an ATM card, bank books and car documents.

Consequently, accused-appellant was charged with violation of Section 16, Article III, Republic Act No. 6425, otherwise known as the Dangerous Drugs Act of 1972, in an information[5] which reads:

That about 10:30 o’clock in the morning of 30 March 1996 at No. 122 SCL M. Hizon St., Kalookan City and within the jurisdiction of this Honorable court, the above-named accused grouping herself together with some other persons whose liabilities are still being determined in a preliminary investigation, conspiring, confederating and mutually helping one another, did then and there, wilfully, unlawfully and feloniously have in their possession, custody and/or control, methamphetamine hydrochloride popularly known as “shabu”, a regulated drug, with a total weight of 230 grams, without the corresponding license and/or prescription to possess, have custody and/or control over the aforesaid regulated drug.

CONTRARY TO LAW.

Accused-appellant filed a motion to quash the search warrant[6] asserting that she and her live-in partner Federico Verona had been leasing an apartment unit at No. 120 M. Hizon Street, District 2, Caloocan City, Metro Manila, since 1995 up to the present as certified by the owner of the apartment unit.

ISSUE: Whether or not III. THE LOWER COURT ERRED IN NOT FINDING THAT THE SEARCH CONDUCTED WAS ILLEGAL AND VIOLATIVE OF ACCUSED’S CONSTITUTIONAL RIGHTS;

IV. THE LOWER COURT ERRED IN NOT ACQUITTING THE ACCUSED AFTER FINDING THAT THE SEARCH WAS INDEED CONDUCTED AT A PLACE DIFFERENT FROM THAT DESCRIBED IN THE SEARCH WARRANT.

RULING:ASPlainly, the basic issue submitted for resolution is the reasonableness of the search conducted by the police officers at accused-appellant’s residence.

The trial court, in upholding the validity of the search, stated that:[10]

Re 3rd argument - the fact that the search warrant in question was served at apartment No. 120 and not at the specific address stated therein which is 122 M. Hizon St., Caloocan City will not by itself render as illegal the search and seizure of subject stuff seized by the operatives pursuant thereto. While it is true that the rationale behind the constitutional and procedural requirements that the search warrant must particularly describe the place to be searched is to the end that no unreasonable search warrant and seizure may not be made (sic) and abuses may not be committed, however, this requirement is not without exception. It is the prevailing rule in our jurisdiction that even a description of the place to be searched is sufficient if the officer with the warrant can with reasonable effort ascertain and identify the place intended (People vs. Veloso, G.R. No. L-23051, Oct. 20, 1925).

Significantly in the case at bar the implementing officer SPO2 Teneros was principally the concerned official who conducted an active surveillance on the Accused and subject house (Surveillance Report, Exh. “9”) and pursued this case by filing the corresponding application for the issuance of a search warrant. Perforce, SPO2 TENEROS was thereby placed in a position enabling him to have prior and personal knowledge of particular house intended in the warrant which definitely refer to no other than the very place where the same was accordingly served.

Accused-appellant, on the other hand, maintains that the search was grossly infirm as the subject search warrant authorized the police authorities to search only No. 122 M. Hizon St., Caloocan

City. However, the actual search was conducted at No. 120 M. Hizon St., Caloocan City.

The basic guarantee to the protection of the privacy and sanctity of a person, his home and his possessions against unreasonable intrusions of the State is articulated in Section 2, Article III of the Constitution, which reads:

THE RIGHT OF THE PEOPLE TO BE SECURE IN THEIR PERSONS, HOUSES, PAPERS, AND EFFECTS AGAINST UNREASONABLE SEARCHES AND SEIZURES OF WHATEVER NATURE AND FOR ANY PURPOSE SHALL BE INVIOLABLE, AND NO SEARCH WARRANT OR WARRANT OF ARREST SHALL ISSUE EXCEPT UPON PROBABLE CAUSE TO BE DETERMINED PERSONALLY BY THE JUDGE AFTER EXAMINATION UNDER OATH OR AFFIRMATION OF THE COMPLAINANT AND THE WITNESSES HE MAY PRODUCE, AND PARTICULARLY DESCRIBING THE PLACE TO BE SEARCHED AND THE PERSONS OR THINGS TO BE SEIZED.

For the validity of a search warrant, the Constitution requires that there be a particular description of “the place to be searched and the persons or things to be seized.” The rule is that a description of a place to be searched is sufficient if the officer with the warrant can, with reasonable effort, ascertain and identify the place intended and distinguish it from other places in the community. Any designation or description known to the locality that leads the officer unerringly to it satisfies the constitutional requirement.[11]

Specifically, the requisites for the issuance of a valid search warrant are: (1) probable cause is present; (2) such presence is determined personally by the judge; (3) the complainant and the witnesses he or she may produce are personally examined by the judge, in writing and under oath or affirmation; (4) the applicant and the witnesses testify on the facts personally known to them; and (5) the warrant specifically describes the place to be searched and the things to be seized.[12]

The absence of any of these requisites will cause the downright nullification of the search warrants. The proceedings upon search warrants must be absolutely legal, for there is not a description of process known to the law, the execution of which is more distressing to the citizen. Perhaps there is none which excites such intense feeling in consequence of its humiliating and degrading effect. The warrants will always be construed strictly

CONSTITUTIONAL LAW II: ARREST, SEARCHES, AND SEIZURES

without, however, going the full length of requiring technical accuracy. No presumptions of regularity are to be invoked in aid of the process when an officer undertakes to justify it.[13]

The application for search warrant filed by SPO2 Teneros requested for authority to search specifically the premises of No. 122 M. Hizon St., Caloocan City. The application was accompanied by a sketch[14]of the area which bears two parallel lines indicated as 10th Avenue drawn vertically on the left-hand side of the paper. Intersecting these lines are two other parallel lines drawn horizontally and indicated as M. Hizon. Above and on the left-hand side of the upper parallel line of the lines identified as M. Hizon, is a square marked as “Basketball Court.” A similar drawing placed near the right-hand side of the upper parallel line is another square marked as “PNR Compound”. Beneath the lower parallel line of the lines marked as “M. Hizon” and right at the center is also a square enclosing an “X” sign marked as “122”, presumably No. 122 M. Hizon St., Caloocan City.

During the hearing for the application of the search warrant, police asset Dante Baradilla described the house to be searched as:

Bale dalawangpalapagpo, semi concrete, color cream naangmgabintana ay may rehasnabakal at sliding at saharap ay may terrace at may sasakyansilanaginagamitsapagdeliver ng shabu.[15]

The trial court then conducted an ocular inspection of the area. It turned out that No. 122 M. Hizon St., Caloocan City was a concrete two-storey residential building with steel-barred windows and a terrace. It was owned by a certain Mr. Joseph Ching. The house, however, bore no house number. The house marked No. 122 M. Hizon St., Caloocan City was actually two houses away from accused-appellant’s house at No. 120 M. Hizon St.

On the other hand, No. 120 Hizon St. was a compound consisting of three apartments enclosed by only one gate marked on the outside as No. 120. The different units within No. 120 Hizon St. were not numbered separately. Accused-appellant rented the third unit from the entrance which was supposedly the subject of the search. The entire compound had an area of approximately ninety (90) square meters. The second unit was located at the back of the first unit and the third unit was at the rear end of the

compound. Hence, access to the third unit from M. Hizon Street was only through the first two units and the common gate indicated as No. 120. The occupants of the premises stated that they commonly use No. 120 to designate their residence.

In People v. Veloso, this Court declared that “even a description of the place to be searched is sufficient if the officer with the warrant can with reasonable effort, ascertain and identify the place intended.”[16]The description of the building in the application for a search warrant in Veloso as well as in the search warrant itself refer to “the building No. 124 CalleArzobispo, City of Manila, Philippine Islands” which was considered “sufficient designation of the premises to be searched.”[17]

The prevailing circumstances in the case at bar are definitely different from those in Veloso. At first glance, the description of the place to be searched in the warrant seems to be sufficient. However, from the application for a search warrant as well as the search warrant itself, the police officer serving the warrant cannot, with reasonable effort, ascertain and identify the place intended precisely because it was wrongly described as No. 122, although it may have been located on the same street as No. 120. Even the description of the house by police asset Baradilla referred to that house located at No. 122 M. Hizon St., not at No. 120 M. Hizon St.

The particularity of the place described is essential in the issuance of search warrants to avoid the exercise by the enforcing officers of discretion. Hence, the trial court erred in refusing to nullify the actions of the police officers who were perhaps swayed by their alleged knowledge of the place. The controlling subject of search warrants is the place indicated in the warrant itself and not the place identified by the police.[18]

It may well be that the police officer identified No. 120 M. Hizon St. as the subject of the actual search. However, as indicated in the witness’ affidavit[19] in support of the application for a search warrant,[20]No. 122 M. Hizon St. was unmistakably indicated. Inexplicably, a few days after the search warrant was issued by the court and served at No. 120 M. Hizon St., SPO2 Teneros informed Judge Bayhon in the return of search warrant[21] that the warrant “was properly served at 122 M. Hizon St., Caloocan City, Metro Manila as indicated in the search warrant itself.”

SPO2 Teneros attempted to explain the error by saying that he thought the house to be searched bore the address 122 M. Hizon St., Caloocan City instead of No. 120 M. Hizon St.[22] But as this Court ruled in Paper Industries Corporation of the Philippines v. Asuncion,[23] thus:

The police had no authority to search the apartment behind the store, which was the place indicated in the warrant, even if they really intended it to be the subject of their application. Indeed, the place to be searched cannot be changed, enlarged or amplified by the police, viz.:

“x xx. In the instant case, there is no ambiguity at all in the warrant. The ambiguity lies outside the instrument, arising from the absence of a meeting of the minds as to the place to be searched between the applicants for the warrant and the Judge issuing the same; and what was done was to substitute for the place that the Judge had written down in the warrant, the premises that the executing officers had in their mind. This should not have been done. It [was] neither fair nor licit to allow police officers to search a place different from that stated in the warrant on the claim that the place actually searched—although not that specified in the warrant—[was] exactly what they had in view when they applied for the warrant and had demarcated in their supporting evidence. What is material in determining the validity of a search is the place stated in the warrant itself, not what the applicants had in their thoughts, or had represented in the proofs they submitted to the court issuing the warrant. Indeed, following the officer’s theory, in the context of the facts of the case, all four (4) apartment units at the rear of Abigail’s Variety Store would have been fair game for a search.

The place to be searched, as set out in the warrant, cannot be amplified or modified by the officers’ own personal knowledge of the premises, or the evidence they adduced in support of their application for the warrant. Such a change is proscribed by the Constitution which requires inter alia the search warrant to particularly describe the place to be searched as well as the persons or things to be seized. It would concede to police officers the power of choosing the place to be searched, even if it not be delineated in the warrant. It would open wide the door to abuse of the search process, and grant to officers executing a search warrant that discretion which the Constitution has precisely removed from them. The particularization of the description of the place to be searched may properly be done only by the Judge,

CONSTITUTIONAL LAW II: ARREST, SEARCHES, AND SEIZURES

and only in the warrant itself; it cannot be left to the discretion of the police officers conducting the search.”

All told, the exclusionary rule necessarily comes into play, to wit:

Art. III, Sec. 3 (2), 1987 Constitution. -- ANY EVIDENCE OBTAINED IN VIOLATION OF THIS OR THE PRECEDING SECTION SHALL BE INADMISSIBLE FOR ANY PURPOSE IN ANY PROCEEDING.

Consequently, all the items seized during the illegal search are prohibited from being used in evidence. Absent these items presented by the prosecution, the conviction of accused-appellant for the crime charged loses its basis.

As the Court noted in an earlier case, the exclusion of unlawfully seized evidence was the only practical means of enforcing the constitutional injunction against unreasonable searches and seizures. Verily, they are the “fruits of the poisonous tree.” Without this exclusionary rule, the constitutional right would be so ephemeral and so neatly severed from its conceptual nexus with the freedom from all brutish means of coercing evidence.[24]

On another note, we find disturbing the variety of the items seized by the searching team in this case. In the return of search warrant, they admitted the seizure of cellular phones, money and television/monitoring device – items that are not within the palest ambit of shabu paraphernalia, which were the only items authorized to be seized. What is more disturbing is the suggestion that some items seized were not reported in the return of search warrant, like the Fiat car, bankbooks, and money. In an attempt to justify the presence of the car in the police station, SPO2 Teneros had to concoct a most incredible story that the accused-appellant, whose pregnancy was already in the third trimester, drove her car to the police station after the intrusion at her house even if the police officers had with them several cars.

A search warrant is not a sweeping authority empowering a raiding party to undertake a fishing expedition to seize and confiscate any and all kinds of evidence or articles relating to a crime.[25]

It need not be stressed anew that this Court is resolutely committed to the doctrine that this constitutional provision is of a

mandatory character and therefore must be strictly complied with. To quote from the landmark American decision of Boyd v. United States: “It is the duty of courts to be watchful for the constitutional rights of the citizen, and against any stealthy encroachments thereon. Their motto should beobstaprincipiis.”[26]

Those who are supposed to enforce the law are not justified in disregarding the rights of the individual in the name of order. Order is too high a price to pay for the loss of liberty. As Justice Holmes declared: “I think it is less evil that some criminals escape than that the government should play an ignoble part.” It is simply not allowed in free society to violate a law to enforce another, especially if the law violated is the Constitution itself.[27]

UNILAB VS ISIP

FACTS: A UNILAB hired a private investigator to investigate a place purported to be manufacturing fake UNILAB products, especially Revicon multivitamins. The agent took some photographs where the clandestine manufacturing operation was taking place. UNILAB then sought the help of the NBI, which thereafter filed an application for the issuance of search warrant in the RTC of Manila. After finding probable cause, the court issued a search warrant directing the police to seize “finished or unfinished products of UNILAB, particularly REVICON multivitamins.” No fake Revicon was however found; instead, sealed boxes where seized, which, when opened contained 60 ml bottles of Disudrin and 200mg tablets of Inoflox, both were brands used by UNILAB. NBI prayed that some of the sized items be turned over to the custody of the Bureau of Food and Drugs (BFAD) for examination. The court granted the motion. The respondents then filed a motion to quash the search warrant or to suppress evidence, alleging that the seized items are considered to be fruit of a poisonous tree, and therefore inadmissible for any purpose in any proceeding, which the petitioners opposed alleging that the boxes of Disudrin and Inoflox were seized under the plain view doctrine. The court, however, granted the motion of the respondents.

ISSUE: Whether or not the seizure of the sealed boxes which, when opened, contained Disudrin syrup and Inoflox, were valid under the plain view doctrine.

RULING: It is true that things not described in the warrant may be seized under the plain view doctrine. However, seized things

not described in the warrant cannot be presumed as plain view. The State must adduce evidence to prove that the elements for the doctrine to apply are present, namely: (a) the executing law enforcement officer has a prior justification for an initial intrusion or otherwise properly in a position from which he can view a particular order; (b) the officer must discover incriminating evidence inadvertently; and (c) it must be immediately apparent to the police that the items they observe may be evidence of a crime, contraband, or otherwise subject to seizureIt was thus incumbent on the NBI and the petitioner to prove that the items were seized on plain view. It is not enough that the sealed boxes were in the plain view of the NBI agents. However, the NBI failed to present any of officers who were present when the warrant was enforced to prove that the the sealed boxes was discovered inadvertently, and that such boxes and their contents were incriminating and immediately apparent. It must be stressed that only the enforcing officers had personal knowledge whether the sealed boxes and their contents thereof were incriminating and that they were immediately apparent. There is even no showing that the NBI agents knew the contents of the sealed boxes before they were opened. In sum then, the petitioner and the NBI failed to prove that the plain view doctrine applies to the seized items.

KATZ VS UNITED STATES

FACTS:The petitioner, Charles Katz, was charged with conducting illegal gambling operations across state lines in violation of federal law. In order to collect evidence against Katz, federal agents placed a warrantless wiretap on the public phone booth that he used to conduct these operations. The agents listened only to Katz's conversations, and only to the parts of his conversations dealing with illegal gambling transactions.

In the case of Olmstead v. United States (1928), the Supreme Court held that the warrantless wiretapping of phone lines did not constitute an unreasonable search under the Fourth Amendment. According to the Court, physical intrusion (a trespass) into a given area, and not mere voice amplification (the normal result of a wiretap), is required for an action to constitute a Fourth Amendment search. This is known as the "trespass doctrine." Partly in response to this decision, Congress passed the Federal Communications Act of 1933. This Act required, among other things, federal authorities to obtain a warrant before wiretapping

CONSTITUTIONAL LAW II: ARREST, SEARCHES, AND SEIZURES

private phone lines. In the case of Silverman v. United States (1961), the Supreme Court refined the Olmsteadtrespass doctrine by holding that an unreasonable search occurs only if a "constitutionally protected area" has been intruded upon.

At his trial, Katz sought to exclude any evidence connected with these wiretaps, arguing that the warrantless wiretapping of a public phone booth constitutes an unreasonable search of a "constitutionally protected area" in violation of the Fourth Amendment. The federal agents countered by saying that a public phone booth was not a "constitutionally protected area," therefore, they could place a wiretap on it without a warrant.

ISSUE:Does the warrantless wiretapping of a public phone booth violate the unreasonable search and seizure clause of the Fourth Amendment to the United States Constitution?

RULING: ASDA YES, By a 7-1 vote, the U.S. Supreme Court agreed with Katz and held that placing of a warrantless wiretap on a public phone booth constitutes an unreasonable search in violation of the Fourth Amendment. The majority opinion, written by Justice Potter Stewart, however, did not address the case from the perspective of a "constitutionally protected area." In essence, the majority argued that both sides in the case were wrong to think that the permissibility of a warrantless wiretap depended upon the area being placed under surveillance. "For the Fourth Amendment protects people, not places. What a person knowingly exposes to the public, even in his own home or office, is not a subject of Fourth Amendment protection . . . . But what he seeks to preserve as private even in an area accessible to the public, may be constitutionally protected," the Court stated.

Building upon this reasoning, the Court held that it was the duty of the Judiciary to review petitions for warrants in instances in which persons may be engaging in conduct that they wish to keep secret, even if it were done in a public place. The Court held that, in the absence of a judicially authorized search warrant, the wiretaps of the public phone booth used by Katz were illegal. Therefore, the evidence against him gathered from his conversations should be suppressed.

CONCURRENCE

Justice John Marshall Harlan's Concurrence: Test for Constitutionally Protected SearchesAlthough he agreed with the majority opinion of the Court, Justice Harlan went further to provide a test for what is a constitutionally protected search. He said it was necessary to clarify when private actions, conducted in a public place, may be constitutionally protected. Expanding upon the general principles enunciated by the majority opinion, Justice Harlan proposed the following two-pronged test to address this issue: "My understanding of the rule that has emerged from prior judicial decisions is that there is a twofold requirement, first that a person have exhibited an actual (subjective) expectation of privacy; and second, that the expectation be one that society is prepared to recognize as 'reasonable.'"

Both the Supreme Court and the lower federal courts have looked to this two-pronged test, and not the majority holding per se, to determine when private actions in public places may be constitutionally protected. In essence, this concurrence has come to be seen as the main point of the Katz decision, and it is the test that, typically, has been used when deciding upon the constitutionality of warrantless wiretaps

Lopez vs. Commissioner of Customs [GR L-27968, 3 December 1975]

Second Division, Fernando (J): 4 concur, 1 took no part

Facts:

M/V JoloLema had been under strict surveillance by the combined team of agents of the NBI, PC, RASAC, and City Police of Davao prior to its apprehension at a private wharf in Batjak, Sasa, Davao City. M/V [JoloLema] was skippered (sic) by Capt. AquilinoPantinople and chartered by Mr. Tomas Velasco. During the period from the latter part of August to September 18, 1966, the said vessel was in Indonesian waters where it loaded copra

and coffee beans from Taruna, Pitta, and Mangenito, all of Indonesia. In its trip to Indonesia it brought various merchandise from the Philippines which were exchanged and/or bartered for copra and coffee beans and subsequently taken to Davao City. Said vessel passed Marore, Indonesia on 18 September 1966. On its a way to Tahuna, Indonesia before proceeding to Davao City where it was apprehended on 19 September 1966. At about 3:00 p.m. of the said day, when the vessel was searched and after Captain Pantinople informed the team that Velasco, the charterer of the vessel, had other documents showing that vessel came from Indonesia carrying smuggled copra and coffee, a combined team of Constabulary and Regional Anti-Smuggling Center operatives headed by Earl Reynolds, Senior NBI Agent of Davao, proceeded to the Velasco's room at the Skyroom Hotel in Davao City, to ask for said document.

Velasco was not inside the hotel room when they entered the room. There are conflicting claims whether the manicurist TeofilaIbañez or whether Velasco's wife, who was allegedly inside the room at that time, voluntarily allowed the police officers to enter; and whether the police officers "forcibly opened luggage’s and boxes from which only several documents and papers were found, then seized, confiscated and took away the same," or whether Mrs. Velasco volunteered to open the suitcases and baggages of Velasco and delivered the documents and things contained therein to Reynolds. The Collector of Customs of Davao seized 1,480 sacks of copra and 86 sacks of coffee from the M/V motor vessel JoloLema. The seizure was declared lawful by the Court of Tax Appeals, and its decision was affirmed by the Supreme Court on 29 November 1974 in Nasiad vs. Court of Tax Appeals (GR L-29318, November 29, 1974, 61 SCRA 238). In the present special civil action for certiorari, prohibition and mandamus; the only question left then is whether the search conducted by a party headed by Reynolds without the search warrant for the hotel room of Velasco, who entered into a contract with Jose G. Lopez, the awardee of such Philippine Reparations Commission vessel, for its operation and use ostensibly for fishing, is violative of such constitutional provision.

Issue:

Whether there was consent on the part of the person who was the occupant of the hotel room then rented by Velasco.

CONSTITUTIONAL LAW II: ARREST, SEARCHES, AND SEIZURES

Held:

There was an attempt on the part of Lopez and Velasco to counteract the force of the recital of the written statement of TeofilaIbañez (allegedly wife of Tomas Velasco) by an affidavit of one Corazon Y. Velasco, who stated that she is the legal wife of Velasco, and another by Velasco himself; reiterating that the person who was present at his hotel room was one TeofilaIbañez, "a manicurist by occupation." If such indeed were the case, then it is much more easily understandable why that person, TeofilaIbañez, who could be aptly described as the wrong person at the wrong place and at the wrong time, would have signified her consent readily and immediately. Under the circumstances, that was the most prudent course of action. It would save her and even Velasco himself from any gossip or innuendo. Nor could the officers of the law be blamed if they would act on the appearances. There was a person inside who from all indications was ready to accede to their request. Even common courtesy alone would have precluded them from inquiring too closely as to why she was there. Under all the circumstances, therefore, it can readily be concluded that there was consent sufficient in law to dispense with the need for a search warrant.

People v. Ramos [GR 85401-02, 4 June 1990]

Third Division, Gutierrez Jr. (J):

Facts:

On 29 November 1982, a civilian informer came to the Narcotics Command Office in Olongapo City and reported that a cigarette vendor by the name of "Mama Rose" (Rosalinda Ramos) was selling marijuana at the corner of 3rd Street and Rizal Avenue in Olongapo City. Tests buys were made using marked money. The

Narcotics Command (NARCOM) team proceeded to the place where appellant was selling cigarettes, and arrested the latter for illegal peddling of marijuana. Ramos was requested to take out the contents of her wallet. The four marked five-peso bills used in the test buys were found among her possessions and were confiscated after the serial numbers were confirmed. Search of Ramos’ stall yielded 20 sticks of marijuana cigarettes in a trash can placed under the small table where Ramos displayed the wares she was selling.

Ramos was thereafter brought to the station. At the station, Ramos executed a statement confessing to her crimes which she swore to before Assistant City Fiscal. The marijuana sticks confiscated were sent to the Philippine Constabulary Crime Laboratory (PCCL) for analysis, and thereafter were confirmed to be marijuana.

The defense contends however that she assented to the invitation of the NARCOM operatives for investigation, after search of her buri bags (which she stores the fruits that she sells) were fruitless. She claimed that she was forced to affix her signature on the four 5-peso bills by one Sgt.Sudiacal, purportedly to be the same money which was used to buy marijuana from her, but which she insists was her money being saved for the rentals. She was later brought to the Fiscal’s Office after investigation, where she signed a document. She claimed she was not assisted by any counsel during the investigation, neither during the time she signed the document at the Fiscal’s Office. Two information were filed against Ramos, one for sale (Criminal Case 5991) and the other for possession of marijuana (Criminal Case 5990). After trial, the RTC Olongapo City (Branch 73) found her guilty beyond reasonable doubt in Criminal Case 5990 for violating Section 8 of RA 6425 and sentenced her to imprisonment of 6 years and 1 day and a fine of P6,000. She was likewise found guilty beyond reasonable doubt in Criminal Case 5991 for violating Section 4 of RA 6425 and

was sentenced to life imprisonment and a fine of P20,000. Ramos sought reversal of the decisions with the Supreme Court.

Issue:

Whether Ramos waived her right against the warrantless search of the trash can, where illegal drugs were found, under her control.

Held:

The trash can (where the contraband were found) was found under the table where her legitimate wares were being sold. Ramos he was the only person who had access to the trash can. The same was under her immediate physical control. She had complete charge of the contents of the trash can under the table to the exclusion of all other persons. In law, actual possession exists when the thing is in the immediate occupancy and control of the party. But this is not to say that the law requires actual possession. In criminal law, possession necessary for conviction of the offense of possession of controlled substances with intent to distribute may be constructive as well as actual. It is only necessary that the defendant must have dominion and control over the contraband. These requirements are present in the situation described, where the prohibited drugs were found inside the trash can placed under the stall owned by Ramos. In fact, the NARCOM agents who conducted the search testified that they had to ask Ramps to stand so that they could look inside the trash can under Ramos' papag. The trash can was positioned in such a way that it was difficult for another person to use the trash can. The trash can was obviously not for use by her customers.

Therefore, the twenty sticks of marijuana are admissible in evidence and the trial court's finding that Ramos is guilty of possession is correct.

Veroy v. Layague [GR 95630, 18 June 1992]

En Banc, Paras (J):

Facts:

CONSTITUTIONAL LAW II: ARREST, SEARCHES, AND SEIZURES

Leopoldo and Ma. Luisa Veroyare husband and wife residing in Davao City. When Veroy was promoted to the position of Assistant Administrator of the Social Security System, sometime in June 1988, he and his family transferred to Quezon City. Their residence in Davao City was left to 2 houseboys, Jimmy Favia and Eric Burgos, who had their assigned quarters at a portion of the premises. The Veroys would occasionally send money to Edna Soquilon for the salary of the said houseboys and other expenses for the upkeep of their house. While the Veroys had the keys to the interior of the house, only the key to the kitchen, where the circuit breakers were located, was entrusted to Edna Soquilon to give her access in case of an emergency. On 12 April 1990, Capt. Reynaldo Obrero of the Talomo Patrol Station, PC/INP raided Veroy’s house in Davao City on information that the said residence was being used as a safe house of rebel soldiers. They were able to enter the yard with the help of the caretakers but did not enter the house since the owner was not present and they did not have a search warrant. Permission was requested by phone to Ma. Luisa Veroy who consented on the condition that the search be conducted in the presence of Major Macasaet. The following day, Capt. Obrero and Maj. Macasaet met at the Veroy’s house to conduct the search pursuant to the authority granted by Ma. Luisa. Capt. Obrero recovered a .45 cal. handgun with a magazine containing 7 live bullets in a black clutch bag inside an unlocked drawer in the children’s room. 3 half-full jute sacks containing printed materials of RAM-SFP were also found in the children’s room. A search of the children’s recreation and study area revealed a big travelling bag containing assorted clothing, a small black bag containing a book entitled “Islamic Revolution Future Path of the Nation”, a road map of the Philippines, a telescope, a plastic bag containing assorted medicines and religious pamphlets was found in the master’s bedroom. Inventory and receipt of seized articles were made. The case was referred for preliminary investigation to the Quezon City Assistant Prosecutor , who was designated Acting Provincial Prosecutor for Davao City by the DOJ through Department Order 88 (16 May 1990). In a resolution dated 6 August1990, the Fiscal recommended the filing of an Information against the Veroys for violation of PD1866 (Illegal Possession of Firearms and Ammunitions in Furtherance of Rebellion). Hence, on 8August 1990, an Information for the said offense was filed by the Office of the City Prosecutor of Davao City before the RTC Davao City). No bail was recommended by the prosecution. The fiscal’s resolution was received by the Veroys on 13 August 1990. The latter filed a motion for bail on the same day which was denied for being premature, as they have not been arrested yet. The Veroys voluntarily surrendered to Gen. PantaleonDumlao, but who refused to receive them on the ground that his office has not received copies of their warrants of arrest. In the meantime, on

15 August 1990, the Veroys were admitted to the St. Luke’s Hospital for various ailments brought about or aggravated by the stress and anxiety caused by the filing of the criminal complaint. On 17 August 1990, Gen. Dumlao granted their request that they be allowed to be confined at the hospital and placed under guard thereat. Upon arraignment on 1 October 1990, the Veroys pleaded not guilty and filed a motion for hospital confinement, which was denied. The court ordered their commitment at the Davao City Rehabilitation Center pending trial on the merits. At the conclusion thereof, the court issued a second order denying their motion for reconsideration. The Veroys were returned to the St. Luke’s Hospital where their physical condition remained erratic. Gen. Dumlao informed the Veroys that he had issued a directive for their transfer from the St. Luke’s Hospital to Camp Crame on the basis of the 2 October 1990

Order. They would proceed with their transfer pursuant to the order of the trial court, unless otherwise restrained by the court. The Veroys filed the petition for certiorari, mandamus and prohibition.

Issue:

Whether the permission granted by ma. Luisa Veroy for ascertaining thereat the presence of alleged “rebel soldiers” include the authority to conduct a room to room search once inside the house.

Held:

The Constitution guarantees the right of the people to be secure in their persons, houses, papers and effects against unreasonable searches and seizures (Article III, Section 2 of the 1987Constitution). However, the rule that searches and seizures must be supported by a valid warrant is not an absolute one. Among the recognized exceptions thereto are: (1) a search incidental to an arrest; (2) a search of a moving vehicle; and (3)

seizure of evidence in plain view (People v. LoHo Wing). The necessity of the permission obtained from Ma. Luisa underlines the recognition of Capt. Obrero of the need of a search warrant to enter the house. The permission granted by was for the purpose of ascertaining thereat the presence of the alleged “rebel” soldiers. The permission did not include any authority to conduct a room to room search once inside the house. The police officers had ample time to procure a search warrant but did not. Warrantless searches were declared illegal because the officials conducting the search had every opportunity to secure a search warrant. The items taken were, therefore, products of an illegal search, violative of their constitutional rights. As such, they are inadmissible in evidence in the criminal actions instituted against them. The offense of illegal possession of firearms is malumprohibitum but it does not follow that the subject thereof is necessarily illegal per se. Motive is immaterial in mala prohibita but the subjects of this kind of offense may not be summarily seized simply because they are prohibited. A search warrant is still necessary. Hence, the rule having been violated and no exception being applicable, the articles seized were confiscated illegally and are therefore protected by the exclusionary principle. They cannot be used as evidence against the Veroys in the criminal action against them for illegal possession of firearms. Besides, assuming that there was indeed a search warrant, still in mala prohibita, while there is no need of criminal intent, there must be knowledge that the same existed. Without the knowledge or voluntariness there is no crime.

People vs. Damaso, GR No. 93516,

Facts: On 18 June 1988, Lt. Candido Quijardo, a Philippine Constabulary officer connected with the 152nd PC Company at Lingayen, Pangasinan, and some companions were sent to verify the presence of CPP/NPA members in Barangay Catacdang, Arellano-Bani, Dagupan City.

In said place, the group apprehended Gregorio Flameniano, Berlina Aritumba, Revelina Gamboa and Deogracias Mayaoa. When interrogated, the persons apprehended revealed that there was an underground safehouse at Gracia Village in Urdaneta,

CONSTITUTIONAL LAW II: ARREST, SEARCHES, AND SEIZURES

Pangasinan. After coordinating with the Station Commander of Urdaneta, the group proceeded to the house in Gracia Village. They found subversive documents, a radio, a 1 x 7 caliber .45 firearm and other items. After the raid, the group proceeded to Bonuan, Dagupan City, and put under surveillance the rented apartment of Rosemarie Aritumba, sister of Berlina Aritumba whom they earlier arrested. They interviewed Luzviminda Morados, a visitor of Rosemarie Aritumba. She stated that she worked with Bernie Mendoza/Basilio Damaso. She guided the group to the house rented by Damaso(@Mendoza). When they reached the house, the group found that it had already vacated by the occupants. Since Morados was hesitant to give the new address of Damaso (@Mendoza), the group looked for the Barangay Captain of the place and requested him to point out the new house rented by Damaso (@Mendoza). The group again required Morados to go with them. When they reached the house, the group saw Luz Tanciangco outside. They told her that they already knew that she was a member of the NPA in the area. At first, she denied it, but when she saw Morados she requested the group to go inside the house.

Upon entering the house, the group, as well as the Barangay Captain, saw radio sets, pamphlets entitled "Ang Bayan," xerox copiers and a computer machine. They also found persons who were companions of Luz Tanciangco (namely, Teresita Calosa, Ricardo Calosa, Marites Calosa, Eric Tanciangco and Luzviminda Morados). The group requested the persons in the house to allow them to look around. When Luz Tanciangco opened one of the rooms, they saw books used for subversive orientation, one M-14 rifle, bullets and ammunitions, Kenwood radio, artificial beard, maps and other items. They confiscated the articles and brought them to their headquarters for final inventory. They likewise brought the persons found in the house to the headquarters for investigation. Said persons revealed that Damaso (@Mendoza) was the lessee of the house and owned the items confiscated therefrom. Thus, Basilio Damaso, was criminally charged.

Upon arraignment, Damaso pleaded not guilty to the crime. The defense counsel interposed his objections to the admissibility of the prosecution's evidence for being illegal for lack of a search warrant; and thereafter, manifested that he was not presenting any evidence for the accused. The trial court rendered its decision, finding Damaso guilty beyond reasonable doubt, sentencing the latter to suffer the penalty of Reclusion Perpetua and to pay the costs of the proceedings. Damaso appealed.

Issue: Whether the warrantless search of Damaso’s house is lawful.

Held: Damaso was singled out as the sole violator of PD 1866, in furtherance of, or incident to, or in connection with the crime of subversion. There is no substantial and credible evidence to establish the fact that the appellant is allegedly the same person as the lessee of the house where the M-14 rifle and other subversive items were found or the owner of the said items. Even assuming for the sake of argument that Damaso is the lessee of the house, the case against him still will not prosper, the reason being that the law enforcers failed to comply with the requirements of a valid search and seizure proceedings. The constitutional immunity from unreasonable searches and seizures, being a personal one cannot be waived by anyone except the person whose rights are invaded or one who is expressly authorized to do so in his or her. The records show that Damaso was not in his house at that time Luz Tanciangco and Luz Morados, his alleged helper, allowed the authorities to enter it. There is no evidence that would establish the fact that Luz Morados was indeed Damaso's helper or if it was true that she was his helper, that Damaso had given her authority to open his house in his absence. While the power to search and seize is necessary to the public welfare, still it must be exercised and the law enforced without transgressing the constitutional rights of the citizens, for the enforcement of no statute is of sufficient importance to justify indifference to the basic principles of government. As a consequence, the search conducted by the authorities was illegal. It would have been different if the situation here demanded urgency which could have prompted the authorities to dispense with a search warrant. But the record is silent on this point. The fact that they came to Damaso's house at nighttime, does not grant them the license to go inside his house.

People vs. Evaristo, GR No. 93828

Facts: Peace officers while patrol, heard burst of gunfire and proceeded to investigate in the house of appellant where they were given permission to enter accidentally discovering the firearms in the latter’s possession. Accused-appellant found guilty of illegal possession of firearms contends that the seizure of the evidence is inadmissible because it was not authorized by a valid warrant.

Issue: Whether or not the evidence obtained without warrant in an accidental discovery of the evidence is admissible.

Held: Yes, the firearms seized was valid and lawful for being incidental to a lawful arrest. An offense was committed in the presence or within the view of an officer, within the meaning of the rule authorizing an arrest without a warrant.

The pertinent rule on the matter is Article III of the Constitution, the relevant portion of which provides:

Sec. 2. The right of the people to be secure in their persons, houses, papers and effects against unreasonable searches and seizures of whatever nature and for any purpose shall be inviolable, and no search warrant or warrant of arrest shall issue except upon probable cause to be determined under oath or affirmation of the complainant and the witnesses he may produce, and particularly describing the place to be searched and the persons or things to be seized.

Sec. 3. (1) .

(2) Any evidence obtained in violation of this or the preceding section shall be inadmissible for any purpose in any proceeding.

It is to be noted that what the above constitutional provisions prohibit are unreasonable searches and seizures. For a search to be reasonable under the law, there must, as a rule, be a search warrant validly issued by an appropriate judicial officer. Yet, the rule that searches and seizures must be supported by a valid search warrant is not an absolute and inflexible rule, for jurisprudence has recognized several exceptions to the search warrant requirement. Amongthese exceptions is the seizure of evidence in plain view, adopted by this jurisdiction from the pronouncements of the United States Supreme Court in Harris vs. U.S. 4 and Coolidge vs. New Hampshire. 5 Thus, it is recognized that objects inadvertently falling in the plain view of an officer who has the right to be in the position to have that view, are subject to seizure and may be introduced in evidence. 6

The records in this case show that Sgt. Romerosa was granted permission by the appellant Evaristo to enter his house. The officer's purpose was to apprehend Rosillo whom he saw had sought refuge therein. Therefore, it is clear that the search for firearms was not Romerosa's purpose in entering the house, thereby rendering his discovery of the subject firearms as inadvertent and even accidental.

People v. Omaweng GR 99050, 2 September 1992

Facts: In the morning of 12 September 1988, PC constables with the Mt. Province PC Command put up a checkpoint at the junction of the roads, one going to Sagada and the other to Bontoc. They

CONSTITUTIONAL LAW II: ARREST, SEARCHES, AND SEIZURES

stopped and checked all vehicles that went through the checkpoint. At 9:15 a.m., they flagged down a cream-colored Ford Fiera (ABT-634) coming from the Bontoc Poblacion and headed towards Baguio. The vehicle was driven by Conway Omaweng and had no passengers.

The Constables (Layong, et.al.) asked permission to inspect the vehicle to which Omaweng acceded to. When they peered into the rear of the vehicle, they saw a travelling bag which was partially covered by the rim of a spare tire under the passenger seat on the right side of the vehicle. They asked permission to see the contents of the bag to which Omaweng consented to. When they opened the bag, they found that it contained 41 plastic packets of different sizes containing pulverized substances. The constable gave a packet to his team leader, who, after sniffing the stuff concluded that it was marijuana. The Constables thereafter boarded the vehicles and proceeded to the Bontoc poblacion to report the incident to the PC Headquarters. The prohibited drugs were surrendered to the evidence custodian.

The PC Forensic Chemist at Camp Dangwa, La Trinidad, Benguet conducted 2 chemistry examinations of the substance contained in the plastic packets taken from appellant and found them to be positive for hashish or marijuana. Omaweng was indicted for the violation of Section 4, Article II of RA 6425 (Dangerous Drugs Act of 1972), as amended, in a criminal complaint filed with the MTC Bontoc, Mountain Province on 12 September 1988. Upon his failure to submit counter-affidavits despite the granting of an extension of time to do so, the court declared that he had waived his right to a preliminary investigation and, finding probable cause against Omaweng, ordered the elevation of the case to the proper court. On 14 November 1988, the Office of the Provincial Fiscal of Mountain Province filed an Information charging Omaweng with the violation of Section 47 Article II of the Dangerous Drugs Act of 1972, as amended. After his motion for reinvestigation was denied by the Provincial Fiscal, Omaweng entered a plea of not guilty during his arraignment on 20 June 1989. During the trial on the merits, the prosecution presented 4 witnesses. Omaweng did not present any evidence other than portions of the Joint Clarificatory Sworn Statement, dated 23 December 1988, of prosecution witnesses Joseph Layong and David Fomocod. On 21 March 1991, the trial court promulgated its Judgment convicting Omaweng of the crime of transporting prohibited drugs. Omaweng appealed to the Supreme Court.

Issue: Whether Omaweng was subjected to search which violates his Constitutional right against unreasonable searches and seizures.

Held: Omaweng was not subjected to any search which may be stigmatized as a violation of his Constitutional right against unreasonable searches and seizures. He willingly gave prior consent to the search and voluntarily agreed to have it conducted on his vehicle and travelling bag. The testimony of the PC Constable (Layung) was not dented on cross-examination or rebutted by Omaweng for he chose not to testify on his own behalf. Omaweng waived his right against unreasonable searches and seizures when he voluntarily submitted to a search or consents to have it made in his person or premises. He is precluded from later complaining thereof right to be secure from unreasonable search may, like every right, be waived and such waiver may be made either expressly or impliedly. Since in the course of the valid search 41 packages of drugs were found, it behooved the officers to seize the same; no warrant was necessary for such seizure.

People v. Barros [GR 90640, 29 March 1994]Third Division, Feliciano (J): 3 concur

Facts: On 6 September 1987, M/Sgt. Francis Yag-as and S/Sgt. James Ayan, both members of the P.C.Mountain Province Command, rode the Dangwa Bus bearing Plate ABZ-242 bound for Sabangan, Mountain Province. Upon reaching Chackchakan, Bontoc, Mountain Province, the bus stopped and both M/Sgt. Yag-as and S/Sgt. Ayan, who were seated at the back, saw Bonifacio Barros carrying a carton, board the bus and seated himself on seat 18 after putting the carton under his seat. Thereafter, the bus continued and upon reaching Sabangan, M/Sgt. Yag-as and S/Sgt. Ayan before they alighted, it being their station, called C2C [Fernando] Bongyao to inspect the carton under seat 18. After C2C Bongyao inspected the carton, he found out that it contained marijuana and he asked the passengers who the owner of the carton was but nobody answered. Thereafter, C2C Bongyao alighted with the carton and S/Sgt. Ayan and C2C Bongyao invited Barros to the detachment for questioning as the latter was the suspected owner of the carton containing marijuana. Upon entering the detachment the carton was opened in the presence of Barros. When Barros denied ownership of the carton of marijuana, the P.C. officers called for the bus conductor who pinpointed to Barros as the owner of the carton of marijuana. Barros was charged with violating Section 4 of RA 6425, as amended (Dangerous Drugs Act of 1972). After trial, the trial court convicted Bonifacio Barros of violation of Section 4 of RA 6425 as amended and sentenced him to suffer the penalty of reclusion perpetua and to pay a fine of P20,000.00. Barros appealed.

Issue: Whether the failure of the carton bearer to object to the search made in the moving vehicle, resulting tohis warrantless arrest, constitutes a waiver.

Held: The general rule is that a search and seizure must be carried out through or with a judicial warrant;otherwise such search and seizure becomes "unreasonable" within the meaning of Section 2, Article III of the 1987 Constitution. The evidence secured thereby — i.e., the "fruits" of the search and seizure — will be inadmissible in evidence "for any purpose in any proceeding." The requirement that a judicial warrant must be obtained prior to the carrying out of a search and seizure is, however, not absolute. There are certain exceptions recognized in our law, one of which relates to the search of moving vehicles. Peace officers may lawfully conduct searches of moving vehicles — automobiles, trucks, etc. — without need of a warrant, it not being practicable to secure a judicial warrant before searching a vehicle, since such vehicle can be quickly moved out of the locality or jurisdiction in which the warrant may be sought. In carrying out warrantless searches of moving vehicles, however, peace officers are limited to routine checks, that is, the vehicles are neither really searched nor their occupants subjected to physical or body searches, the examination of the vehicles being limited to visual inspection. When, however, a vehicle is stopped and subjected to an extensive search, such a warrantless search would be constitutionally permissible only if the officers conducting the search have reasonable or probable cause to believe, before the search, that either the motorist is a law-offender or the contents or cargo of the vehicle are or have been instruments or the subject matter or the proceeds of some criminal offense. The Court has in the past found probable cause to conduct without a judicial warrant an extensive search of moving vehicles in situations where (1) there had emanated from a package the distinctive smell of marijuana; (2) agents of the Narcotics Command ("Narcom") of the Philippine National Police ("PNP") had received a confidential report from informers that a sizeable volume of marijuana would be transported along the route where the search was conducted; (3) Narcom agents were informed or "tipped off" by an undercover "deep penetration" agent that prohibited drugs would be brought into the country on a particular airline flight on a given date; (4) Narcom agents had received information thata Caucasian coming from Sagada, Mountain Province, had in his possession prohibited drugs and when the Narcom agents confronted the accused Caucasian, because of a conspicuous bulge in his waistline, he failed to present his passport and other identification papers when requested to do so; and (5) Narcom

CONSTITUTIONAL LAW II: ARREST, SEARCHES, AND SEIZURES

agents had received confidential information that a woman having the same physical appearance as that of the accused would be transporting marijuana. Herein, there is nothing in the record that any circumstance which constituted or could have reasonably constituted probable cause for the peace officers to search the carton box allegedly owned by Barros. The testimony of the law enforcement officers who had apprehended the accused (M/Sgt. Francis Yag-as and S/Sgt. James Ayan), and who had searched the box in his possession, (C2C Fernando Bongyao), simply did not suggest or indicate the presence of any such probable cause. Further, The accused is not to be presumed to have waived the unlawful search conducted on the occasion of his warrantless arrest "simply because he failed to object." To constitute a waiver, it must appear first that the right exists; secondly, that the person involved had knowledge, actual or constructive, of the existence of such a right; and lastly, that said person had an actual intention to relinquish the right. The fact that the accused failed to object to the entry into his house does not amount to a permission to make a search therein. As the constitutional quaranty is not dependent upon any affirmative act of the citizen, the courts do not place the citizen in the position of either contesting an officer's authority by force, or waiving his constitutional rights; but instead they hold that a peaceful submission to a search or seizure is not a consent or an invitation thereto, but is merely a demonstration of regard for the supremacy of the law. Courts indulge every reasonable presumption against waiver of fundamental constitutional rights and that we do not presume acquiescence in the loss of fundamental rights. Accordingly, the search and seizure of the carton box was equally non-permissible and invalid. The "fruits" of the invalid search and seizure — i.e., the 4) kilos of marijuana — should therefore not have been admitted in evidence against Barros.

People vs. Correa [GR 119246, 30 January 1998]En Banc, Martinez (J): 12 concur

Facts: A week before 18 June 1994, Leonardo Dulay was placed under surveillance by the Police Operativesfrom the Drug Enforcement Unit of the Western Police District Command (DEU-WPDC) on account of confidential and intelligence reports received in said Unit about his drug trafficking around Bambang Street, Tondo, Manila. The police surveillance brought forth positive results and confirmed Dulay's illegal drug trade. On 17 June 1994, operatives were alerted that Dulay would transport and deliver a certain quantity of drugs that night on board a owner-type jeep (FMR948). Thereafter, the operatives, together

with the informer proceeded to A. Bonifacio Street on board 3 vehicles, and inconspicuously parked along the side of North Cemetery and waited for the suspect. The police informant spotted Dulay’s vehicle at 3:00 am. The operatives tailed the subject jeepney until they reached Bambang extension and Jose Abad Santos Avenue, where they accosted the passengers of said jeepney. The team inspected a cylindrical tin can of El Cielo Vegetable Cooking Lard, about two feet high, loaded in the vehicle of the appellants. The can contained 8 bundles of suspected dried marijuana flowering tops wrapped in pieces of paper and plastic tapes. The team seized the suspected contrabands and marked each bundle consecutively. The 3 suspects were brought to the police headquarters at DEU-WPDC for investigation. The packages of suspected marijuana were submitted to the NBI for laboratory analysis to determine their chemical composition. The tests confirmed that the confiscated stuff were positive for marijuana and weighed 16.1789 kilograms. The defense, however, contends that the 3 accused were arrested without warrant in Camarin D, Caloocan City, enroute to Dulay’s house to get the things of his child allegedly rushed previously to the Metropolitan Hospital, for an alleged charge of trafficking on 'shabu,' and were brought to the WPDC headquarters at U.N. Avenue, where they were detained. On 12 July 1994, an Information was filed with the RTC Manila (Branch 35) indicting Antonio Correa y Cayton @ "Boyet," Rito Gunida y Sesante @ "Dodong," and Leonardo Dulay y Santos @ "Boy Kuba" for having violated Section 4, Article II of RA 6425, as amended. When arraigned, the 3 accused pleaded not guilty. After trial and on 3 March 1995, the lower court found the appellants guilty as charged and were sentenced to death and a fine of P10 million.

Issue: Whether the accused are precluded from assailing the warrantless search and seizure, due to waiver ontheir part.

Held: Antonio Correa y Cayton @ "Boyet," Rito Gunida y Sesante @ "Dodong," and Leonardo Dulay ySantos @ "Boy Kuba" are precluded from assailing the warrantless search and seizure when they voluntarily submitted to it as shown by their actuation during the search and seizure. They never protested when the police officer opened the tin can loaded in their vehicle, nor when he opened one of the bundles, nor when they, together with their cargo of drugs and their vehicle, were brought to the police station for investigation and subsequent prosecution. When one voluntarily submits to a search or consents to have it made on his person or premises, he is precluded from later complaining thereof The right to be secure from unreasonable search may, like every right, be waived and such waiver may be made either

expressly or impliedly." Further, they effectively waived their constitutional right against the search and seizure by their voluntary submission to the jurisdiction of the trial court, when they entered a plea of not guilty upon arraignment and by participating in the trial.

Caballes vs. Court of Appeals [GR 136292, 15 January 2002]

Facts: About 9:15 p.m. of 28 June 1989, Sgt. Victorino Noceja and Pat. Alex de Castro, while on a routinepatrol in Barangay Sampalucan, Pagsanjan, Laguna, spotted a passenger jeep unusually covered with "kakawati" leaves. Suspecting that the jeep was loaded with smuggled goods, the two police officers flagged down the vehicle. The jeep was driven by Rudy Caballes y Taiño. When asked what was loaded on the jeep, he did not answer, but he appeared pale and nervous. With Caballes' consent, the police officers checked the cargo and they discovered bundles of 3.08 mm aluminum/galvanized conductor wires exclusively owned by National Power Corporation (NAOCOR). The conductor wires weighed 700 kilos and valued at P55,244.45. Noceja asked Caballes where the wires came from and Caballes answered that they came from Cavinti, a town approximately 8 kilometers away from Sampalucan. Thereafter, Caballes and the vehicle with the high-voltage wires were brought to the Pagsanjan Police Station. Danilo Cabale took pictures of Caballes and the jeep loaded with the wires which were turned over to the Police Station Commander of Pagsanjan, Laguna. Caballes was incarcerated for 7 days in the Municipal jail. Caballes was charged with the crime of theft in an information dated 16 October 1989. During the arraignment, Caballes pleaded not guilty and hence, trial on the merits ensued. On 27 April 1993, Regional Trial Court of Santa Cruz, Laguna rendered judgment, finding Caballes, guilty beyond reasonable doubt of the crime of theft. In a resolution dated 9 November 1998, the trial court denied Caballes' motion for reconsideration. The Court of Appeals affirmed the trial court decision on 15 September 1998. Caballes appealed the decision by certiorari.

Issue: Whether Caballes’ passive submission to the statement of Sgt. Noceja that the latter "will look at thecontents of his vehicle and he answered in the positive" be considered as waiver on Caballes’ part on warrantless search and seizure.

Held: Enshrined in our Constitution is the inviolable right of the people to be secure in their persons andproperties against

CONSTITUTIONAL LAW II: ARREST, SEARCHES, AND SEIZURES

unreasonable searches and seizures, as defined under Section 2, Article III thereof. The exclusionary rule under Section 3(2), Article III of the Constitution bars the admission of evidence obtained in violation of such right. The constitutional proscription against warrantless searches and seizures is not absolute but admits of certain exceptions, namely: (1) warrantless search incidental to a lawful arrest recognized under Section 12, Rule 126 of the Rules of Court and by prevailing jurisprudence; (2) seizure of evidence in plain view; (3) search of moving vehicles; (4) consented warrantless search; (5) customs search;(6) stop and frisk situations (Terry search); and (7) exigent and emergency circumstances. In cases where warrant is necessary, the steps prescribed by the Constitution and reiterated in the Rules of Court must be complied with. In the exceptional events where warrant is not necessary to effect a valid search or seizure, or when the latter cannot be performed except without a warrant, what constitutes a reasonable or unreasonable search or seizure is purely a judicial question, determinable from the uniqueness of the circumstances involved, including the purpose of the search or seizure, the presence or absence of probable cause, the manner in which the search and seizure was made, the place or thing searched and the character of the articles procured. It is not controverted that the search and seizure conducted by the police officers was not authorized by a search warrant. The mere mobility of these vehicles, however, does not give the police officers unlimited discretion to conduct indiscriminate searches without warrants if made within the interior of the territory and in the absence of probable cause. Herein, the police officers did not merely conduct a visual search or visual inspection of Caballes' vehicle. They had to reach inside the vehicle, lift the kakawati leaves and look inside the sacks before they were able to see the cable wires. It thus cannot be considered a simple routine check. Also, Caballes' vehicle was flagged down because the police officers who were on routine patrol became suspicious when they saw that the back of the vehicle was covered with kakawati leaves which, according to them, was unusual and uncommon. The fact that the vehicle looked suspicious simply because it is not common for such to be covered with kakawati leaves does not constitute "probable cause" as would justify the conduct of a search without a warrant. In addition, the police authorities do not claim to have received any confidential report or tipped information that petitioner was carrying stolen cable wires in his vehicle which could otherwise have sustained their suspicion. Philippine jurisprudence is replete with cases where tipped

information has become a sufficient probable cause to effect a warrantless search and seizure. Unfortunately,none exists in the present case. Further, the evidence is lacking that Caballes intentionally surrendered his right against unreasonable searches. The manner by which the two police officers allegedly obtained the consent of Caballes for them to conduct the search leaves much to be desired. When Caballes' vehicle was flagged down, Sgt. Noceja approached Caballes and "told him I will look at the contents of his vehicle and he answered in the positive." By uttering those words, it cannot be said the police officers were asking or requesting for permission that they be allowed to search the vehicle of Caballes. For all intents and purposes, they were informing, nay, imposing upon Caballes that they will search his vehicle. The "consent" given under intimidating or coercive circumstances is no consent within the purview of the constitutional guaranty. In addition, in cases where the Court upheld the validity of consented search, it will be noted that the police authorities expressly asked, in no uncertain terms, for the consent of the accused to be searched. And the consent of the accused was established by clear and positive proof. Neither can Caballes' passive submission be construed as an implied acquiescence to the warrantless search. Casting aside the cable wires as evidence, the remaining evidence on record are insufficient to sustain Caballes' conviction. His guilt can only be established without violating the constitutional right of the accused against unreasonable search and seizure.

PEOPLE VS ASIS

Facts: DaniloAsis y Fonperada and Gilbert Formento y Saricon were charged in an Information dated 18 February 1998; the information stating "That on or about February 10, 1998, in the City of Manila, Philippines, the said accused, conspiring and confederating together and mutually helping each other, did then and there wilfully, unlawfully and feloniously, with intent to gain and by means of force and violence upon person, to wit: by then and there stabbing one YU HING GUAN @ ROY CHING with a bladed instrument on the different parts of the body thereafter take, rob and carry away the following, to wit: Cash money in the amount of P20,000.00; one (1) wristwatch' one (1) gold necklace; and undetermined items; or all in the total amount of P20,000.00 more or less, belonging to said YU HING GUAN @ ROY CHING against his will, to the damage and prejudice of the said owner in the aforesaid amount more or less of P20,000.00, Philippine

Currency, and as a result thereof, he sustained mortal stab wounds which were the direct and immediate cause of his death." When arraigned on 9 July 1998, both accused pleaded not guilty. Found to be deaf-mutes, they were assisted, not only by a counsel de oficio, but also by an interpreter from the Calvary Baptist Church. The prosecution presented 9 witnesses. Although none of them had actually seen the crime committed, strong and substantial circumstantial evidence presented by them attempted to link both accused to the crime. After due trial, both accused were found guilty and sentenced to death. The Regional Trial Court (RTC) of Manila (Branch 54; Criminal Case 98-163090), on 8 March 2000, held that the "crime charged and proved is robbery with homicide under Article 294, No. 1 of the Revised Penal Code," ruled that "although no witnesses to the actual killing and robbery were presented, the circumstantial evidence including the recovery of bloodstained clothing from both accused definitely proved that the two (2) x xx committed the crime," and appreciated the aggravating circumstances of abuse of confidence, superior strength and treachery and thus sentenced both accused to the supreme penalty of death. Hence, the automatic review before the Supreme Court. Both the accused do not question the legality of their arrest, as they made no objection thereto before the arraignment, but object to the introduction of the bloodstained pair of shorts allegedly recovered from the bag of Formento; arguing that the search was illegally done, making the obtainment of the pair of shorts illegal and taints them as inadmissible. The prosecution, on the other hand, contends that it was Formento's wife who voluntarily surrendered the bag that contained the bloodstained trousers of the victim, and thus claims that her act constituted a valid consent to the search without a warrant.

Issue: Whether Formento, a deaf-mute, has given consent to the recovery of the bloodstained pair of short, in his possession during the warrantless search.

Held: Primarily, the constitutional right against unreasonable searches and seizures, being a personal one, cannot be waived by anyone except the person whose rights are invaded or who is expressly authorized to doso on his or her behalf. In the present case, the testimonies of the prosecution witnesses show that at the time the bloodstained pair of shorts was recovered, Formento, together with his wife and mother, was present. Being the very subject of the search, necessarily, he himself should have given consent. Since he was physically present, the waiver could

CONSTITUTIONAL LAW II: ARREST, SEARCHES, AND SEIZURES

not have come from any other person. Lopez vs. Commissioner of Customs does not apply as the accused therein was not present when the search was made. Further, to constitute a valid waiver, it must be shown that first, the right exists; second, the person involved had knowledge, actual or constructive, of the existence of such a right; and third, the person had an actual intention to relinquish the right. Herein, Formento could not have consented to a warrantless search when, in the first place, he did not understand what was happening at that moment. There was no interpreter to assist him -- a deaf-mute -- during the arrest, search and seizure. The point in the case Pasion vda. de Garcia v. Locsin, i.e. "as the constitutional guaranty is not dependent upon any affirmative act of the citizen, the courts do not place the citizen in the position of either contesting an officer’s authority by force, or waiving his constitutional rights; but instead they hold that a peaceful submission to a search or seizure is not a consent or an invitation thereto, but is merely a demonstration of regard for the supremacy of the law," becomes even more pronounced in the present case, in which Formento is a deaf-mute, and there was no interpreter to explain to him what was happening. His seeming acquiescence to the search without a warrant may be attributed to plain and simple confusion and ignorance. The bloodstained pair of shorts was a piece of evidence seized on the occasion of an unlawful search and seizure. Thus, it is tainted and should thus be excluded for being the proverbial fruit of the poisonous tree. In the language of the fundamental law, it shall be inadmissible in evidence for any purpose in any proceeding. Lastly, as to evidence vis-a-is the case in its totality, circumstantial evidence that merely arouses suspicions or gives room for conjecture is not sufficient to convict. It must do more than just raise the possibility, or even the probability, of guilt. It must engender moral certainty. Otherwise, the constitutional presumption of innocence prevails, and the accused deserves acquittal

PEOPLE VS TUDTUD

Facts: Sometime during the months of July and August 1999, the Toril Police Station, Davao City received a report from a “civilian asset” named BobongSolier about a certain Noel Tudtud. Solier related that his neighbors have been complaining about Tudtud, who was allegedly responsible for the proliferation of marijuana in their area. Reacting to the report, PO1 Ronald Desierto, PO1 RamilFloreta and their superior, SPO1 Villalonghan, all members of the Intelligence Section of the Toril Police Station, conducted surveillance in Solier’s neighborhood in Sapa, Toril, Davao City.

For 5 days, they gathered information and learned that Tudtud was involved in illegal drugs. According to his neighbors, Tudtud was engaged in selling marijuana. On 1 August 1999, Solier informed the police that Tudtud had headed to Cotabato and would be back later that day with new stocks of marijuana. Solier described Tudtud as big-bodied and short, and usually wore a hat. At around 4:00 p.m.that same day, a team composed of PO1 Desierto, PO1 Floreta and SPO1 Villalonghan posted themselves at the corner of Saipon and McArthur Highway to await Tudtud’s arrival. All wore civilian clothes. About 8:00 p.m., 2 men disembarked from a bus and helped each other carry a carton marked “King Flakes.” Standing some 5 feet away from the men, PO1 Desierto and PO1 Floreta observed that one of the men fit Tudtud’s description. The same man also toted a plastic bag. PO1 Floreta and PO1 Desierto then approached the suspects and identified themselves as police officers. PO1 Desierto informed them that the police had received information that stocks of illegal drugs would be arriving that night. The man who resembled Tudtud’s description denied that he was carrying any drugs. PO1 Desierto asked him if he could see the contents of the box. Tudtud obliged, saying, “it was alright.” Tudtud opened the box himself as his companion looked on. The box yielded pieces of dried fish, beneath which were two bundles, one wrapped in a striped plastic bag and another in newspapers. PO1 Desierto asked Tudtud to unwrap the packages. They contained what seemed to the police officers as marijuana leaves. The police thus arrested Tudtud and his companion, informed them of their rights and brought them to the police station. The two did not resist. The confiscated items were turned over to the Philippine National Police (PNP) Crime Laboratory

for examination. Forensic tests on specimens taken from the confiscated items confirmed the police officers’ suspicion. The plastic bag contained 3,200 grams of marijuana leaves while the newspapers contained another 890 grams. Noel Tudtud and his companion, DindoBulong, were subsequently charged before the Regional Trial Court (RTC) of Davao City with illegal possession of prohibited drugs. Upon arraignment, both accused pleaded not guilty. The defense, however, reserved their right to question the validity of their arrest and the seizure of the evidence against them. Trial ensued thereafter. Tudtud, denying the charges against them, cried frame-up. Swayed by the prosecution’s evidence beyond reasonable doubt, the RTC rendered judgment convicting both accused as charged and sentencing them to suffer the penalty of reclusion perpetua and to pay a fine of

P500,000.00. On appeal, Noel Tudtud and DindoBolong assign, among other errors, the admission in evidence of the marijuana leaves, which they claim were seized in violation of their right against unreasonable searches and seizures.

Issue: Whether the Tudtud’s implied acquiescence (Tudtud’s statement of “it’s all right” when the police officers requested that the box be opened) be considered a waiver

Held: The right against unreasonable searches and seizures is secured by Section 2, Article III of the Constitution. The RTC justified the warrantless search of appellants’ belongings under the first exception, as a search incident to a lawful arrest. A search incidental to a lawful arrest is sanctioned by the Rules of Court. It is significant to note that the search in question preceded the arrest. Recent jurisprudence holds that the arrest must precede the search; the process cannot be reversed. Nevertheless, a search substantially contemporaneous with an arrest can precede the arrest if the police have probable cause to make the arrest at the outset of the search. The question, therefore, is whether the police herein had probable cause to arrest Tudtud, et. al. The long-standing rule in this jurisdiction, applied with a great degree of consistency, is that “reliable information” alone is not sufficient to justify a warrantless arrest under Section 5 (a), Rule 113. The rule requires, in addition, that the accused perform some overt act that would indicate that he “has committed, is actually committing, or is attempting to commit an offense.” For the exception in Section 5 (a), Rule 113 to apply, this Court ruled, two elements must concur: (1) the person to be arrested must execute an overt act indicating he has just committed, is actually committing, or is attempting to commit a crime; and (2) such overt act is done in the presence or within the view of the arresting officer. Reliable information alone is insufficient. Thus, herein, in no sense can the knowledge of the arresting officers that Tudtud was in possession of marijuana be described as “personal,” having learned the same only from their informant Solier. Solier, for his part, testified that he obtained his information only from his neighbors and the friends of Tudtud.Solier’s information is hearsay. Confronted with such a dubious informant, the police perhaps felt it necessary to conduct their own “surveillance.” This “surveillance,” it turns out, did not actually consist of staking out Tudtud to catch him in the act of plying his illegal trade, but of a mere “gathering of information from the assets there.” The police officers who conducted such “surveillance” did not identify who these “assets” were or the

CONSTITUTIONAL LAW II: ARREST, SEARCHES, AND SEIZURES

basis of the latter’s information. Clearly, such information is also hearsay, not of personal knowledge. Finally, there is an effective waiver of rights against unreasonable searches and seizures only if the following requisites are present: (1) It must appear that the rights exist; (2) The person involved had knowledge, actual or constructive, of the existence of such right; (3) Said person had an actual intention to relinquish the right. Here, the prosecution failed to establish the second and third requisites. Records disclose that when the police officers introduced themselves as such and requested Tudtud that they see the contents of the carton box supposedly containing the marijuana, Tudtud said “it was alright.” He did not resist and opened the box himself. Tudtud's implied acquiescence, if at all, could not have been more than mere passive conformity given under coercive or intimidating circumstances and is, thus, considered no consent at all within the purview of the constitutional guarantee. Consequently, Tudtud's lack of objection to the search and seizure is not tantamount to a waiver of his constitutional right or a voluntary submission to the warrantless search and seizure. As the search of Tudtud's box does not come under the recognized exceptions to a valid warrantless search, the marijuana leaves obtained thereby are inadmissible in evidence. And as there is no evidence other than the hearsay testimony of the arresting officers and their informant, the conviction of Tudtud, et. al. cannot be sustained.

People vs. dela Cruz [GR 83260, 18 April 1990] Second Division, Regalado (J): 4 concur

Facts: After receiving a confidential report from Arnel, their informant, a "buy-bust" operation was conducted by the 13th Narcotics Regional Unit through a team composed of T/Sgt. Jaime Raposas as Team Leader, S/Sgt. RodelitoOblice, Sgt. Dante Yang, Sgt. Vicente Jimenez, P/Pfc. Adolfo Arcoy as poseur-buyer and Pat. DeograciasGorgonia at Maliclic St., Tondo, Manila at around 2:30 p.m. of 4 May 1987 to catch the pusher/s. P/Pfc. Adolfo Arcoy acted as the poseur-buyer with Arnel as his companion to buy marijuana worth P10.00 from the two accused, Juan de la Cruz and Reynaldo Beltran. At the scene, it was Juan de la Cruz whom Arcoy first negotiated with on the purchase and when Arcoy told De la Cruz that he was buying P10.00 worth of marijuana, De la Cruz instructed Reynaldo Beltran to give one aluminum foil of marijuana which Beltran got from his pants' pocket and delivered it to Arcoy. After ascertaining that the foil of suspected marijuana was really marijuana, Arcoy gave the

prearranged signal to his teammates by scratching his head and his teammates who were strategically positioned in the vicinity, converged at the place, identified themselves as NARCOM agents and effected the arrest of De la Cruz and Beltran. The P10.00 marked bill used by Arcoy was found in the possession of Juan de la Cruz together with two aluminum foils and containing marijuana. Juan de la Cruz y Gonzales and Reynaldo Beltran y Aniban were charged in Criminal Case 87-54417 of the Regional Trial Court (RTC) of Manila with violation of Section 4, Art. II, in relation to Section 21, Article IV of Republic Act 6425, as amended. The court, on 15 March 1988, found Dela Cruz and Beltran guilty beyond reasonable doubt and sentenced each of them to suffer the penalty of reclusion perpetua, with the accessory penalties provided by law; to pay a fine of P20,000.00, without subsidiary imprisonment in case of insolvency, and each to pay one-half of the costs. From this decision, de la Cruz and Beltran appealed. In a letter of the Warden, Manila City Jail, dated 3 March 1989, the Court was informed of the death of de la Cruz on 21 February 1989. Thus, the criminal case against de la Cruz was dismissed in the Supreme Court resolution of 25 September 1989. The present appellate proceeding is limited only to Beltran.

Issue: Whether the warrantless seizure incidental to the buy-bust operation violates Beltran’s constitutional rights against unreasonable search and seizure.

Held: A buy-bust operation is the method employed by peace officers to trap and catch a malefactor in flagrante delicto. It is essentially a form of entrapment since the peace officer neither instigates nor induces the accused to commit a crime. Entrapment is the employment of such ways and means for the purpose of trapping or capturing a lawbreaker from whose mind the criminal intent originated. Oftentimes, it is the only effective way of apprehending a criminal in the act of the commission of the offense. While it is conceded that in a buy-bust operation, there is seizure of evidence from one's person without a search warrant, needless to state a search warrant is not necessary, the search being incident to a lawful arrest. A peace officer may, without a warrant, arrest a person when, in his presence, the person to be arrested has committed, is actually committing or is attempting to commit an offense. It is a matter of judicial experience that in the arrest of violators of the Dangerous Drugs Act in a buy-bust operation, the malefactors were invariably caught redhanded. There being no violation of the constitutional right against

unreasonable search and seizure, the confiscated articles are admissible in evidence.

People vs. Tangliben [GR L-63630, 6 April 1990]Third Division, Gutierrez Jr. (J): 4 concur

Facts: In the late evening of 2 March 1982, Patrolmen SilverioQuevedo and Romeo L. Punzalan of the SanFernando Police Station, together with Barangay TanodMacarioSacdalan, were conducting surveillance mission at the Victory Liner Terminal compound located at Barangay San Nicolas, San Fernando, Pampanga. The surveillance mission was aimed not only against persons who may commit misdemeanors at the said place but also on persons who may be engaging in the traffic of dangerous drugs based on informations supplied by informers. Around 9:30 p.m., said Patrolmen noticed a person carrying a red traveling bag who was acting suspiciously and they confronted him. The person was requested by Patrolmen Quevedo and Punzalan to open the red traveling bag but the person refused, only to accede later on when the patrolmen identified themselves. Found inside the bag were marijuana leaves wrapped in a plastic wrapper and weighing one kilo, more or less. The person was asked of his name and the reason why he was at the said place and he gave his name as MedelTangliben and explained that he was waiting for a ride to Olongapo City to deliver the marijuana leaves. The accused was taken to the police headquarters at San Fernando, Pampanga, for further investigation; and that Pat. SilverioQuevedo submitted to his Station Commander his Investigator's Report. The Regional Trial Court, Branch 41, Third Judicial Region at San Fernando, Pampanga, found MedelTangliben y Bernardino guilty beyond reasonable doubt of violating Section 4, Article II of Republic Act 6425 (Dangerous Drugs Act of 1972 as amended) and sentenced him to life imprisonment, to pay a fine of P20,000 and to pay the costs. Tangliben appealed.

Issue: Whether the warrantless search incident to a lawful arrest, even in light of the Court’s ruling in Peoplevs. Aminnudin.

Held: One of the exceptions to the general rule requiring a search warrant is a search incident to a lawfularrest. Thus, Section 12 (Search incident to a lawful arrest) of Rule 126 of the 1985 Rules on Criminal Procedure provides that "A person lawfully arrested may be searched for dangerous weapons or anything which may be used as proof of the

CONSTITUTIONAL LAW II: ARREST, SEARCHES, AND SEIZURES

commission of an offense, without a search warrant." Meanwhile, Rule 113, Sec. 5(a) provides that "A peace officer or a private person may, without a warrant, arrest a person: (a)When, in his presence, the person to be arrested has committed, is actually committing, or is attempting to commit an offense." Tangliben was caught in flagrante, since he was carrying marijuana at the time of his arrest. This case therefore falls squarely within the exception. The warrantless search was incident to a lawful arrest and is consequently valid. The Court is not unmindful of its decision in People v. Aminnudin (163 SCRA 402 [1988]). In that case the PC officers had earlier received a tip from an informer that accused-appellant was on board a vessel bound for Iloilo City and was carrying marijuana. Acting on this tip, they waited for him one evening, approached him as he descended from the gangplank, detained him and inspected the bag he was carrying. Said bag contained marijuana leaves. The Court held that the marijuana could not be admitted in evidence since it was seized illegally, as there was lack of urgency, and thus a search warrant can still be procured. However, herein, the case presented urgency. Although the trial court's decision did not mention it, the transcript of stenographic notes reveals that there was an informer who pointed to Tangliben as carrying marijuana. Faced with such on-the- spot information, the police officers had to act quickly. There was not enough time to secure a search warrant. The Court cannot therefore apply the ruling in Aminnudin herein. To require search warrants during on-the-spot apprehensions of drug pushers, illegal possessors of firearms, jueteng collectors, smugglers of contraband goods, robbers, etc. would make it extremely difficult, if not impossible to contain the crimes with which these persons are associated.

People v. Kalubiran [GR 84079, 6 May 1991]First Division, Cruz (J): 4 concur

Facts: Nestor Kalubiran was arrested on 12 July 1985, in Dumaguete City, by Narcotics Command(NARCOM) elements. His arrest was the result of a "buy-bust" operation in which Pat. Leon Quindo acted asthe buyer while the other team members lay in wait to arrest Kalubiran at the pre- arranged signal. Quindo approached the accused-appellant, who was with a group of friends in front of the Gamo Memorial Clinic, and asked if he could "score," the jargon for

buying marijuana. Kalubiran immediately produced two sticks of marijuana, for which Quindo paid him a previously marked P5.00 bill. Quindo then gave the signal and Cpl. Levi Dorado approached and arrested Kalubiran. Dorado frisked the accused-appellant. He recovered the marked money and found 17 more sticks of marijuana on Kalubiran's person. The other team members, namely M/Sgt. RanulfoVillamor and Sgt. Ruben Laddaran, came later in a jeep, where they boarded Kalubiran to take him to the police station. The 19 sticks of marijuana were marked and then taken to the PC Crime Laboratory, where they were analyzed, with positive results. Kalubiran contended however that one Quindo approached and frisk him on the same night, and found nothing on him. However, he was called back by one Villamor, who told him at gun point to board the jeep and taken to PC headquarters, then to the police station. He was released the following day with the help of a lawyer. After trial, the Regional Trial Court (RTC) Dumaguete City found Kalubiran guilty as charged and sentenced him to life imprisonment plus a P20,000 fine. Kalubiran appealed.

Issue;Whether Kalubiran should be made to answer for the 19 sticks of marijuana found in his possessionduring his arrest.

Held: Kalubiran was arrested in flagrante delicto as a result of the entrapment and so came under Section 5,Rule 113 of the Rules of Court, authorizing a warrantless arrest of any person actually committing a crime. The search was made as an incident of a lawful arrest and so was also lawful under Section 12 of Rule 116. In addition to the Rules, there is abundant jurisprudence justifying warrantless searches and seizures under the conditions established in the case. However, Kalubiran was accused only of selling the two sticks of marijuana under Section 4 of the Dangerous Drugs Act when he should also have been charged with possession of the 17 other sticks found on his person at the time of his arrest. It is unfortunate that he cannot be held to answer for the second offense because he has not been impleaded in a separate information for violation of Section 8 of the said law.

People v. Malmstedt [GR 91107, 19 June 1991]En Banc, Padilla (J): 8 concur, 1 on leave

Facts: Mikael Malmstedt, a Swedish national, entered the Philippines for the 3rd time in December 1988 as atourist. He had visited the country sometime in 1982 and 1985. In the evening of 7 May 1989, Malmstedt left for Baguio City.

Upon his arrival thereat in the morning of the following day, he took a bus to Sagada and stayed in that place for 2 days. On 11 May 1989, Capt. Alen Vasco of NARCOM, stationed at Camp Dangwa, ordered his men to set up a temporary checkpoint at Kilometer 14, Acop, Tublay, Mountain Province, for the purpose of checking all vehicles coming from the Cordillera Region. The order to establish a checkpoint in the said area was prompted by persistent reports that vehicles coming from Sagada were transporting marijuana and other prohibited drugs. Moreover, information was received by the Commanding Officer of NARCOM, that same morning, that a Caucasian coming from Sagada had in his possession prohibited drugs. At about 1:30 pm, the bus where Malmstedt was riding was stopped. Sgt. Fider and CIC Galutan boarded the bus and announced that they were members of the NARCOM and that they would conduct an inspection. During the inspection, CIC Galutan noticed a bulge on Malmstedt's waist. Suspecting the bulge on Malmstedt's waist to be a gun, the officer asked for Malmstedt's passport and other identification papers. When Malmstedt failed to comply, the officer required him to bring out whatever it was that was bulging on his waist, which was a pouch bag. When Malmstedt opened the same bag, as ordered, the officer noticed 4 suspicious-looking objects wrapped in brown packing tape, which turned out to contain hashish, a derivative of marijuana, when opened. Malmstedt stopped to get 2 travelling bags from the luggage carrier, each containing a teddy bear, when he was invited outside the bus for questioning. It was observed that there were also bulges inside the teddy bears which did not feel like foam stuffing. Malmstedt was then brought to the headquarters of the NARCOM at Camp Dangwa for further investigation. At the investigation room, theofficers opened the teddy bears and they were found to also contain hashish. Representative samples were taken from the hashish found among the personal effects of Malmstedt and the same were brought to the PC Crime Laboratory for chemical analysis, which established the objects examined as hashish. Malmstedt claimed that the hashish was planted by the NARCOM officers in his pouch bag and that the 2 travelling bags were not owned by him, but were merely entrusted to him by an Australian couple whom he met in Sagada. He further claimed that the Australian couple intended to take the same bus with him but because there were no more seats available in said bus, they decided to take the next ride and asked Malmstedt to take charge of the bags, and that they would meet each other at the Dangwa Station. An information was filed against Malmstedt for violation of the Dangerous Drugs Act. During the arraignment, Malmstedt

CONSTITUTIONAL LAW II: ARREST, SEARCHES, AND SEIZURES

entered a plea of "not guilty." After trial and on 12 October 1989, the trial court found Malmstedt guilty beyond reasonable doubt for violation of Section 4, Article II of RA 6425 and sentenced him to life imprisonment and to pay a fine of P20,000. Malmstedt sought reversal of the decision of the trial court.

Issue: Whether the personal effects of Malmstedt may be searched without an issued warrant.

Held: The Constitution guarantees the right of the people to be secure in their persons, houses, papers andeffects against unreasonable searches and seizures. However, where the search is made pursuant to a lawful arrest, there is no need to obtain a search warrant. A lawful arrest without a warrant may be made by a peace officer or a private person under the following circumstances. Section 5 provides that “a peace officer or a private person may, without a warrant, arrest a person (a) When, in his presence, the person to be arrested has committed, is actually committing, or is attempting to commit an offense; (b) When an offense has in fact just been committed, and he has personal knowledge of facts indicating that the person to be arrested has committed it; and (c) When the person to be arrested is a prisoner who has escaped from a penal establishment or place where he is serving final judgment or temporarily confined while his case is pending, or has escaped while being transferred from one confinement to another. In cases falling under paragraphs (a) and (b) hereof, the person arrested without a warrant shall be forthwith delivered to the nearest police station or jail, and he shall be proceeded against in accordance with Rule 112, Section 7." Herein, Malmstedt was caught in flagrante delicto, when he was transporting prohibited drugs. Thus, the search made upon his personal effects falls squarely under paragraph (1) of the foregoing provisions of law, which allow a warrantless search incident to a lawful arrest.